You are on page 1of 66

Total Marks : 200

Online Prelims TEST - 19 (SUBJECT WISE)


( InsightsIAS Mock Test Series for UPSC Preliminary Exam 2020 )

1 With reference to Parliamentary Committees, consider the following statements


1. The Rules of Business provide for their establishment
2. The Cabinet can review their decisions.

Which of the statements given above is/are correct?


A. 1 only
B. 2 only
C. Both 1 and 2
D. Neither 1 nor 2

Correct Answer : C

Answer Justification :

The functions of Parliament are not only varied in nature, but considerable in volume. The time at
its disposal is limited. It cannot make very detailed scrutiny of all legislative and other matters that
come up before it. A good deal of Parliamentary business is, therefore, transacted in the
committees. Both Houses of Parliament have a similar committee structure, with a few exceptions.
Their appointment, terms of office, functions and procedure of conducting business are also more or
less similar and are regulated as per rules made by the two Houses under Article 118(1) of the
Constitution. They are extra-constitutional in emergence. In other words, they are not
mentioned in the Constitution. However, the Rules of Business provide for their
establishment

They are an organizational device to reduce the enormous workload of the Cabinet. They also
facilitate in-depth examination of policy issues and effective coordination. They are based on the
principles of division of labour and effective delegation. Hence, statement 1 is correct.

They not only sort out issues and formulate proposals for the consideration of the Cabinet, but also
take decisions. However, the Cabinet can review their decisions. Hence, statement 2 is
correct.

2 With reference to Parliamentary forums, consider the following statements


1. These are constituted for sharing of knowledge and providing a platform of discussions
2. The Speaker is the ex-officio President of all the parliamentary forums.

Which of the statements given above is/are correct?


A. 1 only
B. 2 only
C. Both 1 and 2
D. Neither 1 nor 2


Correct Answer : A

Answer Justification :

1
Total Marks : 200
Online Prelims TEST - 19 (SUBJECT WISE)
( InsightsIAS Mock Test Series for UPSC Preliminary Exam 2020 )

Parliamentary Forums are constituted for sharing of knowledge and providing a platform of
discussions and dialogue between parliamentarians and government officials and key experts.
Hence, statement 1 is correct.

The Speaker is the ex-officio President of all the parliamentary forums except the Parliamentary
Forum on Population and Public Health where the chairman of Rajya Sabha is the ex-officio
President and the Speaker of the Lok Sabha is ex-officio Co-President. Hence, statement 2 is
incorrect.

3 With reference to the acting judge of High Court , consider the following statements:
1. Chief Justice of India(CJI) can appoint a duly qualified person as an acting judge of a high court.
2. An acting judge holds office until the permanent judge resumes his office

Which of the statements given above is/are correct?


A. 1 only
B. 2 only
C. Both 1 and 2
D. Neither 1 nor 2


Correct Answer : C

Answer Justification :

The President can appoint duly qualified persons as additional judges of a high court for a
temporary period not exceeding two years when: there is a temporary increase in the business of
the high court; or there are arrears of work in the high court.

The President can also appoint a duly qualified person as an acting judge of a high court when a
judge of that high court (other than the chief justice) is: unable to perform the duties of his office
due to absence or any other reason; or appointed to act temporarily as chief justice of that high
court. Hence, statement 1 is correct.

An acting judge holds office until the permanent judge resumes his office. However, both the
additional or acting judge cannot hold office after attaining the age of 62 years. Hence, statement
2 is correct.

4 With reference to Writ Jurisdictions, consider the following statements


1. The writ jurisdiction of the high court is exclusive in the state.
2. The writ jurisdiction of the high court is wider than that of the Supreme Court.

Which of the statements given above is/are correct?


A. 1 only
B. 2 only
C. Both 1 and 2
D. Neither 1 nor 2

2
Total Marks : 200
Online Prelims TEST - 19 (SUBJECT WISE)
( InsightsIAS Mock Test Series for UPSC Preliminary Exam 2020 )


Correct Answer : B

Answer Justification :

The Constitution has constituted the Supreme Court as the guarantor and defender of the
fundamental rights of the citizens. The Supreme Court is empowered to issue writs including
habeas corpus, mandamus, certiorari, prohibition and quo-warrento for the enforcement of the
fundamental rights of the citizens and for any other purpose. The phrase ‘for any other purpose’
refers to the enforcement of an ordinary legal right

The writ jurisdiction of the high court (under Article 226) is not exclusive but concurrent
with the writ jurisdiction of the Supreme Court (under Article 32). It means, when the
fundamental rights of a citizen are violated, the aggrieved party has the option of moving either the
high court or the Supreme Court directly. Hence, statement 1 is incorrect.

The writ jurisdiction of the high court is wider than that of the Supreme Court. This is
because, the Supreme Court can issue writs only for the enforcement of fundamental rights and not
for any other purpose, and that is, it does not extend to a case where the breach of an ordinary legal
right is alleged. Hence, statement 2 is correct.

5 With reference to Electronic Voting Machine (EVM), consider the following statements
1. EVMs do not require electricity for its functioning.
2. An EVM being used by Election Commission of India can record a maximum of 2,00,000 votes.
3. EVMs were first used in Karnataka assembly elections.

Which of the statements given above is/are correct?


A. 2 only
B. 1 only
C. 2 and 3 only
D. 1, 2 and 3

Correct Answer : B

Answer Justification :

Electronic Voting Machine (EVM) is an electronic device for recording votes. An Electronic Voting
Machine consists of two Units – a Control Unit and a Balloting Unit – joined by a five-meter cable.
The Control Unit is placed with the Presiding Officer or a Polling Officer and the Balloting Unit is
placed inside the voting compartment. Instead of issuing a ballot paper, the Polling Officer in-
charge of the Control Unit will release a ballot by pressing the Ballot Button on the Control Unit.
This will enable the voter to cast his vote by pressing the blue button on the Balloting Unit against
the candidate and symbol of his choice.

EVMs do not require electricity. EVMs run on an ordinary battery assembled by Bharat Electronics
Limited/Electronics Corporation of India Limited. Hence, statement 1 is correct.

3
Total Marks : 200
Online Prelims TEST - 19 (SUBJECT WISE)
( InsightsIAS Mock Test Series for UPSC Preliminary Exam 2020 )

An EVM being used by ECI can record a maximum of 2,000 votes. Hence, statement 2 is
incorrect.

EVMs were first used in 70-Parur Assembly Constituency of Kerala in the year 1982. Hence,
statement 3 is incorrect.

6 With reference to Electoral bonds, consider the following statements


1. It will not carry name of payee.
2. Electoral Bonds may be purchased by only citizen of India.
3. Only registered political parties, that have secured not less than 1% of votes polled in last election
of Lok Sabha or legislative assembly of state will be eligible to receive electoral bonds.

Which of the statements given above is/are correct?


A. 2 only
B. 1 and 3 only
C. 2 and 3 only
D. 1, 2 and 3

Correct Answer : D

Answer Justification :

Electoral Bond is a financial instrument (similar to a promissory note) for making donations to
political parties. These are issued by Scheduled Commercial banks upon authorisation from the
Central Government to intending donors, but only against cheque and digital payments (it cannot be
purchased by paying cash). These bonds shall be redeemable in the designated account of a
registered political party within the prescribed time limit from issuance of bond.

These electoral bonds can be bought for any value in multiples of Rs 1,000, Rs 10,000, Rs 1 lakh, Rs
10 lakh or Rs 1 crore after fulfilling all existing Know Your Customer (KYC) norms and making
payments from bank account. It will not carry name of payee. The bond deposited by any eligible
political party to its account shall be credited on the same day. Hence, statement 1 is correct.

Eligible political parties can encash electoral bonds only through their bank accounts. Electoral
Bonds may be purchased by only citizen of India. Hence, statement 2 is correct.

Only registered political parties, that have secured not less than 1% of votes polled in last election
of Lok Sabha or legislative assembly of state, will be eligible to receive electoral bonds. Hence,
statement 3 is correct.

7 With reference to Indian Parliamentary Group, consider the following statements


1. The Indian Parliamentary Group is an autonomous body.
2. Chairman of Rajya sabha is its ex-officio President

Which of the statements given above is/are correct?


A. 1 only

4
Total Marks : 200
Online Prelims TEST - 19 (SUBJECT WISE)
( InsightsIAS Mock Test Series for UPSC Preliminary Exam 2020 )

B. 2 only
C. Both 1 and 2
D. Neither 1 nor 2

Correct Answer : A

Answer Justification :

The Indian Parliamentary Group is an autonomous body, membership of which is open to all current
or former members of the Indian Parliament. Hence, statement 1 is correct.

Speaker of Lok Sabha is its ex-officio President. It was founded in 1949, following a Motion adopted
by the Constituent Assembly on 16 August 1948. Hence, statement 2 is incorrect.

Membership of the Indian Parliamentary Group is open to all Members of Parliament and ex-
Members of Parliament. A Member of Parliament can become a life Member of the Group on
payment of life subscription. On ceasing to be a Member of Parliament, a life member of the Group
is designated as ‘Associate Life Member’. The management and control of the affairs of the Group
are vested in the Executive Committee. The Speaker, Lok Sabha is the ex-officio President of the
Group.

8 With reference to Model Code of Conduct (MCC), consider the following statements
1. Mosques, Churches, Temples or other places of worship shall not be used as forum for election
propaganda.
2. The MCC is enforceable by law.

Which of the statements given above is/are correct?


A. 1 only
B. 2 only
C. Both 1 and 2
D. Neither 1 nor 2

Correct Answer : A

Answer Justification :

Election Commission of India's Model Code of Conduct is a set of guidelines issued by the Election
Commission of India for conduct of political parties and candidates during elections mainly with
respect to speeches, polling day, polling booths, portfolios, election manifestos, processions and
general conduct. These set of norms has been evolved with the consensus of political parties who
have consented to abide by the principles embodied in the said code in its letter and spirit.

There shall be no appeal to caste or communal feelings for securing votes. Mosques,
Churches, Temples or other places of worship shall not be used as forum for election
propaganda. Hence, statement 1 is correct.

5
Total Marks : 200
Online Prelims TEST - 19 (SUBJECT WISE)
( InsightsIAS Mock Test Series for UPSC Preliminary Exam 2020 )

Its main purpose is to ensure that ruling parties, at the Centre and in the States, do not misuse their
position of advantage to gain an unfair edge. It is designed to avert practices which are deemed
corrupt under model code of conduct. For example, politicians should not make hate speeches,
putting one community against another or make promises about new projects that may sway a
voter.

The MCC is not enforceable by law. However, certain provisions of the MCC may be enforced
through invoking corresponding provisions in other statutes such as the Indian Penal Code, 1860,
Code of Criminal Procedure, 1973, and Representation of the People Act, 1951. Hence, statement
2 is incorrect.

9 With reference to Proxy voting, consider the following statements


1. Under this a registered elector can delegate his/her voting power to a representative nominated by
the returning officer of his/her constituency.
2. Only a “classified service voter” is allowed to nominate a proxy to cast vote on his/her behalf.

Which of the statements given above is/are correct?


A. 1 only
B. 2 only
C. Both 1 and 2
D. Neither 1 nor 2

Correct Answer : B

Answer Justification :

The government of India has provisions for allowing its citizens to cast vote during an election in
three different ways: In person, by post, and through a proxy.

Proxy voting allows a registered elector to delegate his voting rights to a representative he
nominates. This was introduced in 2003 for very few countries to have such a provision. Hence,
statement 1 is incorrect.

Only “classified service voter” serving in the armed forces or paramilitary forces is allowed to avail
of these benefits; the proxy, too, must be a registered voter of the same constituency. Hence,
statement 2 is correct.

10 With reference to Representation of Peoples Act 1951, consider the following statements
1. All the seats in the House of the People allotted to the States shall be filled by the persons chosen
by direct election from parliamentary constituencies in the States.
2. Every State to which only one seat is allotted shall form one parliamentary constituency.

Which of the statements given above is/are correct?


A. 1 only
B. 2 only
C. Both 1 and 2

6
Total Marks : 200
Online Prelims TEST - 19 (SUBJECT WISE)
( InsightsIAS Mock Test Series for UPSC Preliminary Exam 2020 )

D. Neither 1 nor 2

Correct Answer : C

Answer Justification :

Representation of Peoples Act 1951:

The Representation of People Act, 1951 is an act of Parliament of India to provide for the
conduct of elections of the Houses of Parliament and to the House or Houses of the
Legislature of each State, the qualifications and disqualifications for membership of those
Houses, the corrupt practices and other offences at or in connection with such elections and
the decision of doubts and disputes arising out of or in connection with such elections.

The Act was enacted by the provisional parliament under Article 327 of Indian Constitution,
before the first general election.

All the seats in the House of the People allotted to the States shall be seats to be filled by
persons chosen by direct election from parliamentary constituencies in the States. Hence,
statement 1 is correct.

Every parliamentary constituency shall be a single member constituency.

Every State to which only one seat is allotted shall form one parliamentary constituency.
Hence, statement 2 is correct.

11 With reference to political parties registered and recognized by the Election Commission of India,
consider the following statements
1. The recognized parties need one proposer for filing the nomination.
2. Recognized parties are allowed to have forty “star campaigners” during the time of elections.
3. The travel expenses of these star campaigners are included in the election expenditure of the
candidates of their parties.

Which of the statements given above is/are correct?


A. 2 only
B. 1 and 2 only
C. 2 and 3 only
D. 1, 2 and 3

Correct Answer : B

Answer Justification :

7
Total Marks : 200
Online Prelims TEST - 19 (SUBJECT WISE)
( InsightsIAS Mock Test Series for UPSC Preliminary Exam 2020 )

The Election Commission registers political parties for the purpose of elections and grants them
recognition as national or state parties on the basis of their poll performance. The other parties are
simply declared as registered-unrecognized parties.

The recognized parties need only one proposer for filing the nomination. Hence, statement 1 is
correct

Also, these parties are allowed to have forty “star campaigners” during the time of elections and the
registered–unrecognized parties are allowed to have twenty “star campaigners”. Hence,
statement 2 is correct

The travel expenses of these star campaigners are not included in the election expenditure of the
candidates of their parties. Hence, statement 3 is incorrect.

12 With reference to Electoral system in India, consider the following statements


1. Parliament may make provision with respect to all the matters relating to elections to the
Parliament as well as the State legislatures.
2. There is to be only one general electoral roll for every territorial constituency for election to the
Parliament and State legislature.
3. No person can claim to be included in any special electoral roll for any constituency on grounds of
religion, race, caste or sex or any of them.
4. State legislatures can make provision only to supplement the parliamentary law and cannot override
it.

Which of the statements given above is/are correct?


A. 1 and 2 only
B. 2 and 3 only
C. 1, 2 and 4 only
D. 1, 2, 3 and 4


Correct Answer : D

Answer Justification :

All statements given above are correct.

Articles 324 to 329 in Part XV of the Constitution make the following provisions with regard to the
electoral system in our country.

The Constitution (Article 324) provides for an independent Election Commission in order to
ensure free and fair elections in the country. The power of superintendence, direction and
conduct of elections to the Parliament, the state legislatures, the office of the President and
the office of the Vice-President is vested in the Commission. At present, the commission
consists of a chief election commissioner and two election commissioners.

8
Total Marks : 200
Online Prelims TEST - 19 (SUBJECT WISE)
( InsightsIAS Mock Test Series for UPSC Preliminary Exam 2020 )

There is to be only one general electoral roll for every territorial constituency for election to
the Parliament and the state legislatures. Thus, the Constitution has abolished the system of
communal representation and separate electorates which led to the partition of the country.
Hence, statement 2 is correct

No person can claim to be included in any special electoral roll for any constituency on
grounds only of religion, race, caste or sex or any of them. Thus, the Constitution has
accorded equality to every citizen in the matter of electoral franchise. Hence, statement 3 is
correct

Parliament may make provision with respect to all matters relating to elections to the
Parliament and the state legislatures including the preparation of electoral rolls, the
delimitation of constituencies and all other matters necessary for securing their due
constitution. Hence, statement 1 is correct.

The state legislatures can also make provision with respect to all matters relating to elections
to the state legislatures including the preparation of electoral rolls and all other matters
necessary for securing their due constitution.

But, state can make provision for only those matters which are not covered by the Parliament.
In other words, they can only supplement the parliamentary law and cannot override it.
Hence, statement 4 is correct.

13 With reference to Judicial Review, consider the following statements


1. In India, the Constitution confers the power of judicial review on both Supreme Court and High
Court.
2. The power of judicial review can be curtailed or excluded through a constitutional amendment.

Which of the statements given above is/are correct?


A. 1 only
B. 2 only
C. Both 1 and 2
D. Neither 1 nor 2


Correct Answer : A

Answer Justification :

Judicial review is the power of the judiciary to examine the constitutionality of legislative
enactments and executive orders of both the Central and State governments. The doctrine of
judicial review originated and developed in the USA. In India, the Constitution confers the power of
judicial review on the judiciary (both the Supreme Court as well as High Courts). Hence, statement
1 is correct.

9
Total Marks : 200
Online Prelims TEST - 19 (SUBJECT WISE)
( InsightsIAS Mock Test Series for UPSC Preliminary Exam 2020 )

The Supreme Court has declared the power of judicial review as a basic feature of the Constitution
or an element of the basic structure of the Constitution. Hence, the power of judicial review cannot
be curtailed or excluded even by a constitutional amendment. Hence, statement 2 is incorrect.

14 Which of the following committees have not been set up to examine electoral system in India?

A. Dinesh Goswami Committee.


B. Indrajit Gupta Committee.
C. Narsimhan Committee.
D. Tankha Committee.

Correct Answer : C

Answer Justification :

Various committees have been put in place to examine electoral system, election machinery and
election process from 1974 to 2011. The various committees can be mentioned here as follows:

1974 Tarkunde committee appointed by JP Narayana - Election process and election system.

1990 Dinesh Goswami Committee - Electoral reforms.

1993 Vohra committee - Nexus between crime and politics.

1998 Indrajit Gupta committee - State funding of elections.

1999 Law commission of India - Reform of the electoral laws.

2000-2002 National commission headed by MN Venkatachaliah - Working of the constitution.

2004 Election commission of India - Proposed electoral reforms.

2007 Second ARC headed by Verappa Moily - Report on ethics in governance.

2010 Tankha committee - Gamut of election laws and electoral reforms.

Narsimhan Committee is on Banking Sector Reforms (not on electoral reforms). Hence, option (c)
is correct.

10
Total Marks : 200
Online Prelims TEST - 19 (SUBJECT WISE)
( InsightsIAS Mock Test Series for UPSC Preliminary Exam 2020 )

15 With reference to Public Interest Litigation, consider the following statements


1. In a PIL, there is no determination on adjudication of individual rights.
2. It helps in expanding the meaning of fundamental right to equality, life and personal liberty.

Which of the statements given above is/are correct?


A. 1 only
B. 2 only
C. Both 1 and 2
D. Neither 1 nor 2

Correct Answer : C

Answer Justification :

PIL is brought before the Court not for the purpose of enforcing the right of one individual against
another as happens in the case of ordinary litigation, but it is intended to promote and vindicate
public interest. In a PIL, unlike traditional dispute resolution mechanism, there is no determination
on adjudication of individual rights. Hence, statement 1 is correct.

It creates a new regime of human rights by expanding the meaning of fundamental right to equality,
life and personal liberty. Hence, statement 2 is correct.

16 With reference to Totalitarian system, consider the following statements


1. It is a political system in which the state recognizes no limits to its authority.
2. Citizens have only the rights that the ruler chooses to give them.

Which of the statements given above is/are correct?


A. 1 only
B. 2 only
C. Both 1 and 2
D. Neither 1 nor 2


Correct Answer : C

Answer Justification :

Totalitarianism is a political system in which the state recognizes no limits to its authority and
strives to regulate every aspect of public and private life wherever feasible, without any respect for
human rights. A system of government where the rulers have total unchecked control of the nation.
Hence, statement 1 is correct.

A totalitarian regime attempts to control virtually all aspects of the social life, including the
economy, education, art, science, private life, and morals of citizens. The totalitarian government
seeks to completely control the thoughts and actions of its citizens.

Citizens have only the rights that the ruler chooses to give them. This type of power is usually taken
11
Total Marks : 200
Online Prelims TEST - 19 (SUBJECT WISE)
( InsightsIAS Mock Test Series for UPSC Preliminary Exam 2020 )

by force. . Hence, statement 2 is correct.

17 With reference to VVPAT (Voter Verifiable Paper Audit Trail) machines, consider the following
statements.
1. The VVPAT generates a paper slip which has the name of the voter, candidate voted for and the
symbol of his party.
2. It provides means to audit the stored electronic results.

Which of the statements given above is/are correct?


A. 1 only
B. 2 only
C. Both 1 and 2
D. Neither 1 nor 2


Correct Answer : B

Answer Justification :

A VVPAT is a machine that is attached to the Electronic Voting Machine (EVM). A voter casts his or
her vote on the EVM as is currently being done. The VVPAT attached to the EVM generates a paper
slip which has the name of the candidate voted for and the symbol of his party. Hence, statement
1 is incorrect.

A VVPAT is intended as an independent verification system for voting machines designed to allow
voters to verify that their vote was cast correctly, to detect possible election fraud or malfunction,
and to provide a means to audit the stored electronic results. Hence, statement 2 is correct.

18 With reference to special officer for linguistic minorities, consider the following statements
1. He is appointed by the prime minister.
2. Its report is laid before each house of parliament.

Which of the statements given above is/are correct?


A. 1 only
B. 2 only
C. Both 1 and 2
D. Neither 1 nor 2


Correct Answer : B

Answer Justification :

Under Article 350B, a special officer for linguistic minorities has been enshrined in the constitution.
This officer is to be appointed by the president. Hence, statement 1 is incorrect.

Its job is to investigate all matters relating to the safeguards provided for linguistic minorities and

12
Total Marks : 200
Online Prelims TEST - 19 (SUBJECT WISE)
( InsightsIAS Mock Test Series for UPSC Preliminary Exam 2020 )

report to the president. This report is also one of those reports laid before each house of parliament
and sent to the government of the states concerned. Hence, statement 2 is correct.

19 With reference to Fast Track Courts, consider the following statements


1. It is the primary responsibility of the State Governments to establish these courts in consultation
with the concerned High Courts.
2. Maharashtra is the first state to get funds from Central government for setting up of FTC.

Which of the statements given above is/are correct?


A. 1 only
B. 2 only
C. Both 1 and 2
D. Neither 1 nor 2

Correct Answer : A

Answer Justification :

On the recommendation of 14th Finance Commission, a scheme for setting up of Fast Track Courts
(FTC) was rolled out by the central government. They were established to expeditiously dispose of
long pending cases in the Sessions Courts and long pending cases of under trial prisoners. It is the
primary responsibility of the State Governments to establish these courts in consultation with the
concerned High Courts. Hence, statement 1 is correct.

U.P is first on the list of beneficiary states to get funds from Central government for setting up of
FTC, followed by Maharashtra. Hence, statement 2 is incorrect

20 With reference to Delimitation Commission of India, consider the following statements


1. It is a statutory body
2. The Decisions made the body cannot be question by any court of law.

Which of the statements given above is/are correct?


A. 1 only
B. 2 only
C. Both 1 and 2
D. Neither 1 nor 2

Correct Answer : C

Answer Justification :

The Delimitation commission or Boundary commission of India is a commission established by the


Government of India under the provisions of the Delimitation Commission Act. The main task of the
commission is redrawing the boundaries of the various assembly and Lok Sabha constituencies
based on a recent census. Hence, statement 1 is correct.

13
Total Marks : 200
Online Prelims TEST - 19 (SUBJECT WISE)
( InsightsIAS Mock Test Series for UPSC Preliminary Exam 2020 )

Decisions made the body cannot be question by any court of law. Hence, statement 2 is correct.

21 Consider the following statements


1. The governor is indirectly elected by a specially constituted electoral college same like president.
2. The office of governor of a state is considered as an employment under the Central government.

Which of the statements given above is/are correct?


A. 1 only
B. 2 only
C. Both 1 and 2
D. Neither 1 nor 2

Correct Answer : D

Answer Justification :

The governor is neither directly elected by the people nor indirectly elected by a specially
constituted electoral college as is the case with the president. He is appointed by the president by
warrant under his hand and seal. Hence, statement 1 is incorrect.

As held by the Supreme Court in 1979, the office of governor of a state is not an employment under
the Central government. It is an independent constitutional office and is not under the control of or
subordinate to the Central government. Hence, statement 2 is incorrect.

22 Consider the following statements


1. The Chief Minister shall be appointed by the Governor
2. The Constitution contains specific procedure for the selection and appointment of the Chief
Minister.

Which of the statements given above is/are correct?


A. 1 only
B. 2 only
C. Both 1 and 2
D. Neither 1 nor 2


Correct Answer : A

Answer Justification :

Article 164 only says that the Chief Minister shall be appointed by the governor. However, this does
not imply that the governor is free to appoint any one as the Chief Minister. In accordance with the
convections of the parliamentary system of government, the governor has to appoint the leader of
the majority party in the state legislative assembly as the Chief Minister. Hence, statement 1 is
correct.

14
Total Marks : 200
Online Prelims TEST - 19 (SUBJECT WISE)
( InsightsIAS Mock Test Series for UPSC Preliminary Exam 2020 )

The Constitution does not contain any specific procedure for the selection and appointment of the
Chief Minister. Hence, statement 2 is incorrect.

23 With reference to State Council of Ministers, consider the following statements


1. The salaries and allowances of ministers shall be determined by Governor.
2. The advice tendered by Ministers to the Governor shall be inquired only by Supreme Court of India.
3. The council of ministers shall be collectively responsible to the state Legislative Assembly.

Which of the statements given above is/are correct?


A. 3 only
B. 1 and 3 only
C. 2 and 3 only
D. 1, 2 and 3


Correct Answer : A

Answer Justification :

The council of ministers in the states is constituted and function in the same way as the council of
ministers at the Centre. The salaries and allowances of ministers shall be determined by the state
legislature. Hence, statement 1 is incorrect.

The advice tendered by Ministers to the Governor shall not be inquired into in any court. Hence,
statement 2 is incorrect.

The council of ministers shall be collectively responsible to the state Legislative Assembly. Hence,
statement 3 is correct.

24 With reference to the Speaker of State Legislative Assembly, consider the following statements
1. He decides whether a bill is a Money Bill or not
2. He appoints the chairmen of all the committees of the assembly.

Which of the statements given above is/are correct?


A. 1 only
B. 2 only
C. Both 1 and 2
D. Neither 1 nor 2


Correct Answer : C

Answer Justification :

The Speaker of Assembly maintains order and decorum in the assembly for conducting its business
and regulating its proceedings. This is his primary responsibility and he has final power in this
regard. He decides whether a bill is a Money Bill or not and his decision on this question is final.

15
Total Marks : 200
Online Prelims TEST - 19 (SUBJECT WISE)
( InsightsIAS Mock Test Series for UPSC Preliminary Exam 2020 )

Hence, statement 1 is correct.

He appoints the chairmen of all the committees of the assembly and supervises their functioning.
He himself is the chairman of the Business Advisory Committee, the Rules Committee and the
General Purpose Committee. Hence, statement 2 is correct.

25 Consider the following statements


1. A Money Bill can be introduced in the legislative council.
2. The legislative assembly and legislative council both have same powers with regard to a money bill.

Which of the statements given above is/are correct?


A. 1 only
B. 2 only
C. Both 1 and 2
D. Neither 1 nor 2


Correct Answer : D

Answer Justification :

The Constitution lays down a special procedure for the passing of Money Bills in the state
legislature. A Money Bill cannot be introduced in the legislative council. It can be introduced in the
legislative assembly only and that too on the recommendation of the governor. Every such bill is
considered to be a government bill and can be introduced only by a minister. Hence, statement 1
is incorrect.

If the legislative council does not return the bill to the legislative assembly within 14 days, the bill is
deemed to have been passed by both Houses at the expiry of the said period in the form originally
passed by the legislative assembly. Thus, the legislative assembly has more powers than legislative
council with regard to a money bill. At the most, the legislative council can detain or delay a money
bill for a period of 14 days. Hence, statement 2 is incorrect.

26 With reference to Lok Adalats, consider the following statements


1. Lok Adalats have been given statutory status under the Legal Services Authorities Act, 1987.
2. Lok Adalats can pass an award only on the consent of parties involved.

Which of the statements given above is/are correct?


A. 1 only
B. 2 only
C. Both 1 and 2
D. Neither 1 nor 2

Correct Answer : A

Answer Justification :

16
Total Marks : 200
Online Prelims TEST - 19 (SUBJECT WISE)
( InsightsIAS Mock Test Series for UPSC Preliminary Exam 2020 )

Lok Adalat is a forum where the cases (or disputes) which are pending in a court or which are at
pre-litigation stage (not yet brought before a court) are compromised or settled in an amicable
manner.

Lok Adalats have been given statutory status under the Legal Services Authorities Act, 1987. Under
the said Act, the award (decision) made by the Lok Adalats is deemed to be a decree of a civil court
and is final and binding on all parties and no appeal against such an award lies before any court of
law. Hence, statement 1 is correct.

A permanent Lok Adalat can pass an award on merits, even without the consent of parties. Such an
award is final and binding. From that no appeal is possible. Hence, statement 2 is incorrect.

27 With reference to Gram Nyayalayas, consider the following statements


1. The Gram Nyayalaya shall be bound by the rules of evidence provided in the Indian Evidence Act,
1872
2. The Gram Nyayalaya shall exercise the powers of both Criminal and Civil Courts.

Which of the statements given above is/are correct?


A. 1 only
B. 2 only
C. Both 1 and 2
D. Neither 1 nor 2

Correct Answer : B

Answer Justification :

The Gram Nyayalayas Act, 2008 has been enacted to provide for the establishment of the Gram
Nyayalayas at the grass roots level for the purposes of providing access to justice to the citizens at
their doorsteps and to ensure that opportunities for securing justice are not denied to any citizen
due to social, economic or other disabilities.

The Gram Nyayalaya shall not be bound by the rules of evidence provided in the Indian Evidence
Act, 1872 but shall be guided by the principles of natural justice and subject to any rule made by
the High Court. Hence, statement 1 is incorrect.

The Gram Nyayalaya shall be a mobile court and shall exercise the powers of both Criminal and
Civil Courts. Hence, statement 2 is correct.

28 With reference to Central Council of Local Government, consider the following statements
1. It was constituted under Article 263 of the Constitution of India.
2. It deals with matters of urban local government only.

Which of the statements given above is/are correct?


A. 1 only
B. 2 only

17
Total Marks : 200
Online Prelims TEST - 19 (SUBJECT WISE)
( InsightsIAS Mock Test Series for UPSC Preliminary Exam 2020 )

C. Both 1 and 2
D. Neither 1 nor 2

Correct Answer : C

Answer Justification :

The Central Council of Local Government was set up in 1954. It was constituted under Article 263
of the Constitution of India by an order of the President of India. Originally, it was known as the
Central Council of Local Self-Government. The Council is an advisory body. It consists of the
Minister for Urban Development in the Government of India and the ministers for local self-
government in states. The Union minister acts as the Chairman of the Council. Hence, statement
1 is correct.

Till 1958, it dealt with both urban as well as rural local governments, but after 1958 it has been
dealing with matters of urban local government only. Hence, statement 2 is correct.

29 With reference to Union Territories, consider the following statements


1. Every union territory is administered by the President acting through an administrator appointed by
him.
2. The Parliament can make laws on any subject of the three lists of Schedule VII for the union
territories.

Which of the statements given above is/are correct?


A. 1 only
B. 2 only
C. Both 1 and 2
D. Neither 1 nor 2


Correct Answer : C

Answer Justification :

Articles 239 to 241 in Part VIII of the Constitution deal with the union territories. Even though all
the union territories belong to one category, there is no uniformity in their administrative system.

Every union territory is administered by the President acting through an administrator


appointed by him. An administrator of a union territory is an agent of the President and not head
of state like a governor. The President can specify the designation of an administrator; it may be
Lieutenant Governor or Chief Commissioner or Administrator. Hence, statement 1 is correct.

The Parliament can make laws on any subject of the three lists (including the State List) for the
union territories. This power of Parliament also extends to Puducherry and Delhi, which have their
own local legislatures. This means that, the legislative power of Parliament for the union territories
on subjects of the State List remain unaffected even after establishing a local legislature for them.
Hence, statement 2 is correct.

18
Total Marks : 200
Online Prelims TEST - 19 (SUBJECT WISE)
( InsightsIAS Mock Test Series for UPSC Preliminary Exam 2020 )

30 With reference to Criteria for the status of Classical Language, the language should be
1. High antiquity of its early texts/recorded history over a period of 1500-2000 years.
2. A body ancient literature/texts, which is considered a valuable heritage by generations of speakers.
3. The literary tradition is original and not borrowed from another speech community.
4. The classical language and literature being distinct from modern, there may also be a discontinuity
between language and its later forms or its offshoots.

Which of the statements given above is/are correct?


A. 1 and 2 only
B. 2 and 3 only
C. 1, 2 and 4 only
D. 1, 2, 3 and 4


Correct Answer : D

Answer Justification :

A classical language is a language with a literature that is classical. The languages are generally
taken to have a "classical" stage. Such a stage is limited in time and is considered "classical" if it
comes to be regarded as a literary "golden age" retrospectively. The Government of India in 2004
declared that languages to meet certain requirements could be accorded the status of a 'Classical
Languages in India'.

Criteria for the status of a 'Classical Languages in India'

High antiquity of its early texts/recorded history over a period of 1500-2000 years. Hence,
statement 1 is correct.

A body ancient literature/texts, which is considered a valuable heritage by generations of


speakers. Hence, statement 2 is correct.

The literary tradition is original and not borrowed from another speech community. Hence,
statement 3 is correct.

The classical language and literature being distinct from modern, there may also be a
discontinuity between language and its later forms or its offshoots. Hence, statement 4 is
correct.

31 With reference to Armed Forces Tribunal (AFT), consider the following statements
1. Its judicial member should have been a judge of Supreme Court.
2. Armed Forces Tribunal (AFT) is considered to be a criminal court with respect to Indian Penal Code,
and Code of Criminal Procedure.

19
Total Marks : 200
Online Prelims TEST - 19 (SUBJECT WISE)
( InsightsIAS Mock Test Series for UPSC Preliminary Exam 2020 )

Which of the statements given above is/are correct?


A. 1 only
B. 2 only
C. Both 1 and 2
D. Neither 1 nor 2

Correct Answer : B

Answer Justification :

Armed Forces Tribunal is a military tribunal in India. It was established under the Armed Forces
Tribunal Act, 2007. In 1999, the Law Commission's 169th report stated that disciplinary and service
matters required quick resolutions and proposed a special tribunal for the military and paramilitary
forces. A judicial member should have been a judge of High Court. Hence, statement 1 is
incorrect.

AFT is considered to be a criminal court with respect to Indian Penal Code, and Code of Criminal
Procedure. Hence, statement 2 is correct.

32 With reference to National Company Law Tribunal (NCLT), consider the following statements
1. The Central Government has constituted NCLT under the Companies Act, 2013.
2. It is a quasi-judicial body.

Which of the statements given above is/are correct?


A. 1 only
B. 2 only
C. Both 1 and 2
D. Neither 1 nor 2

Correct Answer : C

Answer Justification :

The National Company Law Tribunal is a quasi-judicial body in India that adjudicates issues relating
to Indian companies. Hence, statement 2 is correct.

The tribunal was established under the Companies Act 2013 and was constituted on 1 June 2016 by
the government of India and is based on the recommendation of the Justice Jain committee on law
relating to insolvency and winding up of companies. Hence, statement 1 is correct.

33 The Padmanabhaiah Committee is related to

A. Electoral reform
B. Banking reforms
C. Police reforms

20
Total Marks : 200
Online Prelims TEST - 19 (SUBJECT WISE)
( InsightsIAS Mock Test Series for UPSC Preliminary Exam 2020 )

D. Industrial reforms


Correct Answer : C

Answer Justification :

The Padmanabhaiah Committee on Police Reforms (The Committee) was set up by the Ministry of
Home Affairs, Government of India in January 2000. In addition to the Chairman, a former Union
Home Secretary, the Committee consisted of four members, who were all policemen- two retired
and two serving. Hence, option (c) is correct.

34 With reference to Curative Petition, consider the following statements


1. The concept of curative petition was first evolved by the Supreme Court of India in the matter of
Minerva Mills case.
2. No time limit is given for filing Curative petition.

Which of the statements given above is/are correct?


A. 1 only
B. 2 only
C. Both 1 and 2
D. Neither 1 nor 2

Correct Answer : B

Answer Justification :

Curative Petition is the last judicial resort available for redressal of grievances in court which is
normally decided by judge’s in-chamber. It is only in rare cases that such petitions are given an
open-court hearing.

The concept of curative petition was first evolved by the Supreme Court of India in the matter of
Rupa Ashok Hurra vs. Ashok Hurra and Anr. (2002) where the question was whether an aggrieved
person is entitled to any relief against the final judgement/order of the Supreme Court, after
dismissal of a review petition. Hence, statement 1 is incorrect.

No time limit is given for filing Curative petition. Hence, statement 2 is correct.

35 Which of the following directive principles were added by the 42nd Amendment Act of 1976 ?
1. To promote equal justice and to provide free legal aid to the poor.
2. To take steps to secure the participation of workers in the management of industries
3. To protect and improve the environment and to safeguard forests and wild life

Select the correct answer using the code given below:


A. 1 and 2 only
B. 2 and 3 only

21
Total Marks : 200
Online Prelims TEST - 19 (SUBJECT WISE)
( InsightsIAS Mock Test Series for UPSC Preliminary Exam 2020 )

C. 1 and 3 only
D. 1, 2 and 3


Correct Answer : D

Answer Justification :

All the statements given above are correct.

The 42nd Amendment Act of 1976 added four new Directive Principles to
the original list. They require the State:
1. To secure opportunities for healthy development of children (Article 39).
2. To promote equal justice and to provide free legal aid to the poor (Article 39 A).
3. To take steps to secure the participation of workers in the management of
industries (Article 43 A).
4. To protect and improve the environment and to safeguard forests and wild
life (Article 48 A).

36 Consider the following statements:


1. Under the Government of India Act 1919, irrigation was made a Provincial subject.
2. Cabinet Secretary is the chairman of National Water Resource Council.
3. Minor Irrigation census is a centrally sponsored scheme with 100% central funding.

Which of the statements given above is/are correct?


A. 1 only
B. 1 and 3 only
C. 2 only
D. 2 and 3 only

Correct Answer : B

Answer Justification :

Under the Government of India Act 1919, irrigation became a Provincial subject and the
Government of India’s responsibility was confined to advice, co-ordination and settlement of
disputes over right on the water of Inter-Provincial Rivers. Hence, statement 1 is correct.

National Water Resources Council was constituted under the Chairmanship of Prime
Minister. The National Water Resources Council (NWRC) adopted the National Water Policy in
September 1987. National Water Board was constituted in September, 1990 with Secretary,
Ministry of Water Resources as Chairman and Chief Secretaries of all the States / UTs, Secretaries
of concerned Union Ministries and Chairman, Central Water Commission, as Members, in order to
review the progress of implementation of the stipulations of the National Water Policy for reporting
to the NWRC and also initiate effective measures for systematic development of the country’s water
resources. Hence, statement 2 is incorrect.

22
Total Marks : 200
Online Prelims TEST - 19 (SUBJECT WISE)
( InsightsIAS Mock Test Series for UPSC Preliminary Exam 2020 )

The Centrally Sponsored scheme - Rationalization of Minor Irrigation Statistics (RMlS) was
launched in 1987-88 and is being implemented by Minor Irrigation (Stat.) Wing of the Ministry
through State Governments. It is now renamed as "Irrigation Census" which is a Centrally
Sponsored scheme with 100% Central funding. Hence, statement 3 is correct.

The objective of the scheme is to build a comprehensive and reliable database in the Minor
Irrigation Sector for effective planning and policy making. Five Minor Irrigation Censuses with
reference years 1986-87, 1993-94, 2000-01, 2006-07 and 2013-14 have been conducted so far.

The major activities under the scheme are (i) conduct of 6th Minor Irrigation Census with reference
year 2017-18 and (ii) conduct of a Census of Water Bodies which is taken up for the first
time.

http://mowr.gov.in/about-us/history

37 Consider the following statements:


1. Directorate General of Civil Aviation (DGCA) is the regulatory body governing the safety aspects of
civil aviation in India.
2. Licensing of pilots is one of the functions of Directorate General of Civil Aviation (DGCA).

Which of the statements given above is/are correct?


A. 1 only
B. 2 only
C. Both 1 and 2
D. Neither 1 nor 2

Correct Answer : C

Answer Justification :

Both statements given above are correct.

Directorate General of Civil Aviation is the regulatory body governing the safety aspects of
civil aviation in India.

Licensing of pilots, aircraft maintenance engineers, air traffic controllers and flight engineers,
and conducting examinations and checks for that purpose are some of the functions of Directorate
General of Civil Aviation.

http://www.dgca.nic.in/dgca/func-ind.htm

38 Consider the following statements:


1. Biotechnology Industry Research Assistance Council (BIRAC) is a not-for-profit public sector
enterprise (PSE), set up by Department of Biotechnology (DBT).
2. Students Innovations for Translation & Advancement of Research Explorations (SITARE) Scheme is
aimed at supporting innovative student projects in the area of information technology (IT).

23
Total Marks : 200
Online Prelims TEST - 19 (SUBJECT WISE)
( InsightsIAS Mock Test Series for UPSC Preliminary Exam 2020 )

3. LEAP Fund (Launching Entrepreneurial Driven Affordable Products) is an initiative of NITI Aayog.

Which of the statements given above is/are correct?


A. 1 only
B. 2 and 3 only
C. 1 and 2 only
D. 3 only

Correct Answer : A

Answer Justification :

Biotechnology Industry Research Assistance Council (BIRAC) is a not-for-profit Section 8,


Schedule B, Public Sector Enterprise, set up by Department of Biotechnology (DBT),
Government of India as an Interface Agency to strengthen and empower the emerging Biotech
enterprise to undertake strategic research and innovation, addressing nationally relevant product
development needs. Hence, statement 1 is correct.

Students Innovations for Translation & Advancement of Research Explorations (SITARE)


Scheme is aimed at supporting innovative student projects in the area of biotechnology.
The scheme’s mandate is to promote and encourage young students for embracing translational
research (defined under Section 4) to develop innovative products and technologies addressing
unmet needs. SITARE-GYTI Award Grant is not a usual research fellowship. The scheme also
provides an opportunity for orientation, training and mentoring through residential workshops etc.
Hence, statement 2 is incorrect.

While the Bioincubators are able to support the “Space, Services and Knowledge” requirements of
startups, wide gap exists in financial support required by a technology driven start up in the
initial phase. BIRAC’s initiative - LEAP Fund (Launching Entrepreneurial Driven
Affordable Products) primarily aims to address this need through the Incubators.

The LEAP Fund is to provide funding support of up to Rs 1 crore per startup. BioNEST
Bioincubators (LEAP fund partners) will be given up to Rs 5 crore fund/ partner for investing in
potential Startup against equity and equity linked instruments. Hence, statement 3 is incorrect.

39 Consider the following statements:


1. Central Board of Film Certification (CBFC) is a non-statutory body set up by executive resolution.
2. Films can be publicly exhibited in India only after they have been certified by the Central Board of
Film Certification.

Which of the statements given above is/are NOT correct?


A. 1 only
B. 2 only
C. Both 1 and 2
D. Neither 1 nor 2

24
Total Marks : 200
Online Prelims TEST - 19 (SUBJECT WISE)
( InsightsIAS Mock Test Series for UPSC Preliminary Exam 2020 )

Correct Answer : A

Answer Justification :

Central Board of Film Certification (CBFC) is a statutory body under Ministry of Information
and Broadcasting, regulating the public exhibition of films under the provisions of the
Cinematograph Act 1952. Hence, statement 1 is incorrect.

Films can be publicly exhibited in India only after they have been certified by the Central
Board of Film Certification. Hence, statement 2 is correct.

The Board consists of non-official members and a Chairman (all of whom are appointed by
Central Government) and functions with headquarters at Mumbai.

40 Consider the following statements:


1. Article 309 empowers the Parliament to regulate the recruitment and the conditions of service of
the persons appointed to public services and posts under the Centre.
2. A civil servant can be dismissed or removed by an authority subordinate to that by which he was
appointed if he proved guilty in heinous crimes.

Which of the statements given above is/are NOT correct?


A. 1 only
B. 2 only
C. Both 1 and 2
D. Neither 1 nor 2


Correct Answer : B

Answer Justification :

Articles 308 to 314 in part XIV of the Constitution contain provisions with
regard to all-India services, Central services and state services.

Article 309 empowers the Parliament and the state legislatures to regulate the
recruitment and the conditions of service of the persons appointed to public services and
posts under the Centre and the states, respectively. Until such laws are made, the president or
the governor can make rules to regulate these matters. Hence, statement 1 is correct.

Article 311 provides two safeguards to civil servants against any arbitrary
dismissal from their posts:
(a) A civil servant cannot be dismissed or removed by an authority
subordinate to that by which he was appointed. Hence, statement 2 is incorrect.
(b) A civil servant cannot be dismissed or removed or reduced in rank except after an
inquiry in which he has been informed of the charges against him and given a reasonable
opportunity of being heard in respect
of those charges.

25
Total Marks : 200
Online Prelims TEST - 19 (SUBJECT WISE)
( InsightsIAS Mock Test Series for UPSC Preliminary Exam 2020 )

The above two safeguards are available only to the members of the civil services of the Centre, the
all-India services, the civil services of a state or to persons holding civil posts under the Centre or a
state and not to the members of defence services or persons holding military posts

41 Consider the following statements:


1. The Tribes Advisory Councils have been constituted only in the Scheduled Areas States.
2. It shall be the duty of the Tribes Advisory Council to advise on such matters pertaining to the
welfare and advancement of the Scheduled Tribes in the State as may be referred to them by the
Chief Minister of concerned state.

Which of the statements given above is/are correct?


A. 1 only
B. 2 only
C. Both 1 and 2
D. Neither 1 nor 2

Correct Answer : D

Answer Justification :

The Tribes Advisory Councils have been constituted in the Scheduled Areas States i.e. Andhra
Pradesh, Chhattisgarh, Gujarat, Jharkhand, Himachal Pradesh, Madhya Pradesh, Maharashtra,
Odisha and Rajasthan and non-Scheduled Areas States of Tamil Nadu and West
Bengal. Hence, statement 1 is incorrect.

With regard to the Tribes Advisory Council, the Para 4 (1) of the Fifth Schedule to the Constitution
envisages that “There shall be established in each State having Scheduled Areas therein and, if the
President so directs, also in any State having Scheduled Tribes but not Scheduled Areas therein. In
terms of clause (2) of Para 4 of the Fifth Schedule, it shall be the duty of the Tribes
Advisory Council to advise on such matters pertaining to the welfare and advancement of
the Scheduled Tribes in the State as may be referred to them by the Governor. Hence,
statement 2 is incorrect.

42 As per SEBI Regulations, 2015 with reference to the issue of Municipal bonds, which of the following
conditions/criteria has to be met by a municipality or a Corporate Municipal Entity (CME)?
1. The ULB should not have negative net worth in any of ten immediately preceding financial years.
2. The municipality should not have defaulted in repayment of debt securities or loans obtained from
banks or financial institutions during the last five years.
3. The corporate municipal entity, its promoter, group company or director(s), should not have been
named in the list of the wilful defaulters published by the RBI .

Select the correct answer using the code given below:


A. 1 only
B. 3 only
C. 2 and 3 only
D. 1, 2 and 3

26
Total Marks : 200
Online Prelims TEST - 19 (SUBJECT WISE)
( InsightsIAS Mock Test Series for UPSC Preliminary Exam 2020 )

Correct Answer : B

Answer Justification :

Municipal bonds are bonds issued by urban local bodies- municipal bodies and municipal corporates
(entities owned by municipal bodies) to raise money for financing specific projects specifically
infrastructure projects. These bonds are attracting attention as the ULBs urgently need money to
finance infrastructural expenditure. Especially, smart cities and other urban development projects
necessitates them to create finance.

SEBI Guidelines on municipal bonds: Which ULB can issue muni bonds?

As per the SEBI Regulations, 2015, a municipality or a Corporate Municipal Entity (CME) should
meet certain conditions:

The ULB should not have negative net worth in any of three immediately preceding
financial years. Hence, statement 1 is incorrect.

Non-default: The municipality should not have defaulted in repayment of debt


securities or loans obtained from banks or financial institutions during the last 365
days. Hence, statement 2 is incorrect.

Now wilful defaulter: The corporate municipal entity, its promoter, group company or
director(s), should not have been named in the list of the wilful defaulters published by the
RBI or should not have defaulted of payment of interest or repayment of principal amount in
respect of debt instruments issued by it to the public, if any. Hence, statement 3 is correct.

43 With reference to the None Of The Above (NOTA) provision in the ballot papers / EVMs in India,
consider the following statements:
1. It was introduced according to the directions of Supreme Court.
2. The voters polled against the NOTA option are not taken into account for calculating the total valid
voters polled by the contesting candidates for the purpose of return of security deposits to
candidates.

Which of the statements given above is/are correct?


A. 1 only
B. 2 only
C. Both 1 and 2
D. Neither 1 nor 2

Correct Answer : C

Answer Justification :

27
Total Marks : 200
Online Prelims TEST - 19 (SUBJECT WISE)
( InsightsIAS Mock Test Series for UPSC Preliminary Exam 2020 )

Both statements given above are correct.

According to the directions of Supreme Court, the Election Commission made provision in
the ballot papers / EVMs for None of the Above (NOTA) option so that the voters who come to
the polling booth and decide not to vote for any of the candidates in the fray, are able to exercise
their right not to vote for such candidates while maintaining the secrecy of their ballot.

The voters polled against the NOTA option are not taken into account for
calculating the total valid voters polled by the contesting candidates for the purpose of
return of security deposits to candidates. Even if the number of electors opting for NOTA
options is more than the number of votes polled by any of the candidates, the candidate
who secures the largest number of votes has to be declared elected.

44 What is/are the benefits received by the political parties recognised as National and/or State party by
the Election Commission of India?
1. These parties are allowed to have forty “star campaigners” during the time of elections.
2. Provision of time for political broadcasts on the state-owned television and radio stations
3. Access to the electoral rolls.

Select the correct answer using the codes given below


A. 2 only
B. 3 only
C. 1 and 3 only
D. 1, 2 and 3
Telegram Channel
https://t.me/visionpt3652019
Correct Answer : D

Answer Justification :

The recognition granted by the Election Commission to the parties determines their right to
certain privileges like allocation of the party symbols, provision of time for political
broadcasts on the state-owned television and radio stations and access to electoral rolls.

Further, the recognized parties need only one proposer for filing the nomination. Also, these
parties are allowed to have forty “star campaigners” during the time of elections and the
registered–unrecognized parties are allowed to have twenty “star campaigners”. The travel
expenses of these star campaigners are not included in the election expenditure of the
candidates of their parties.

45 Which of the following is/are the functions of National Human Rights Commission (NHRC):
1. To intervene in any proceeding involving allegation of violation of human rights pending before a
court.
2. To visit jails and detention places to study the living conditions of inmates and make
recommendation thereon.
3. To encourage the efforts of non-governmental organisations (NGOs) working in the field of human
rights.

28
Total Marks : 200
Online Prelims TEST - 19 (SUBJECT WISE)
( InsightsIAS Mock Test Series for UPSC Preliminary Exam 2020 )

4. To review the constitutional and other legal safeguards for the protection of human rights and
recommend measures for their effective implementation.

Select the correct answer using the codes given below


A. 1, 2 and 3 only
B. 1, 3 and 4 only
C. 1, 2 and 4 only
D. 1, 2, 3 and 4

Correct Answer : D

Answer Justification :

All statements given above are correct.

The National Human Rights Commission is a statutory but non-constitutional body. It was
established in 1993 under a legislation enacted by the Parliament, namely, the Protection of Human
Rights Act, 1993

The functions of the National Human Rights Commission (NHRC) are:


1. To inquire into any violation of human rights or negligence in the prevention of such violation by
a public servant, either suo motu or on a petition presented to it or on an order of a court.
2. To intervene in any proceeding involving allegation of violation of human rights pending
before a court. 3. To visit jails and detention places to study the living conditions of
inmates and make recommendation thereon.
4. To review the constitutional and other legal safeguards for the protection of human
rights and recommend measures for their effective implementation.
5. To review the factors including acts of terrorism that inhibit the enjoyment of human rights and
recommend remedial measures.
6. To study treaties and other international instruments on human rights and make
recommendations for their effective implementation.
7. To undertake and promote research in the field of human rights 8. To spread human rights
literacy among the people and promote awareness of the safeguards available for the protection of
these rights.
9. To encourage the efforts of non-governmental organisations (NGOs) working in the field
of human rights.
10. To undertake such other functions as it may consider necessary for the promotion of human
rights.

46 Which of the following statements is/are correct about the Inter-State Water Disputes Act, 1956?
1. It has been enacted by the provision under Article 262 of the Indian Constitution.
2. It empowers the Central government to set up an ad hoc tribunal for the adjudication of a dispute
between two or more states in relation to the waters of an inter-state river or river valley.
3. The decision of the tribunal can be appealed only on the Supreme Court of India.

Select the correct answer using the codes given below

29
Total Marks : 200
Online Prelims TEST - 19 (SUBJECT WISE)
( InsightsIAS Mock Test Series for UPSC Preliminary Exam 2020 )

A. 1 and 2 only
B. 3 only
C. 2 and 3 only
D. 1, 2 and 3

Correct Answer : A

Answer Justification :

Article 262 of the Constitution provides for the adjudication of inter-state


water disputes. It makes two provisions:
1. Parliament may by law provide for the adjudication of any dispute or complaint with respect to
the use, distribution and control of waters of any inter-state river and river valley.
2. Parliament may also provide that neither the Supreme Court nor any other court is to exercise
jurisdiction in respect of any such dispute or complaint.
Under this provision, the Parliament has enacted two laws: (a) the River Boards Act (1956) and (b)
the Inter-State Water Disputes Act (1956). Hence, statement 1 is correct.
The Inter-State Water Disputes Act empowers the Central government to set up an ad hoc
tribunal for the adjudication of a dispute between two or more states in relation to the
waters of an inter-state river or river valley. Hence, statement 2 is correct.

The decision of the tribunal would be final and binding on the parties to the dispute.
Neither the Supreme Court nor any other court is to have jurisdiction in respect of any water
dispute which may be referred to such a tribunal under this Act. Hence, statement 3 is incorrect.

47 Consider the following statements:


1. The legislature of a state may adopt more than one language as official language of that particular
state.
2. Official Language Act, 1963 provides for the continued use of English in addition to Hindi, for all
official purposes of the Union and also for the transaction of business in Parliament.

Which of the statements given above is/are correct?


A. 1 only
B. 2 only
C. Both 1 and 2
D. Neither 1 nor 2

Correct Answer : C

Answer Justification :

Both statements given above are correct.

Part XVII of the Constitution deals with the official language in Articles 343 to 351. Its provisions
are divided into four heads—Language of the Union, Regional languages, Language of the judiciary
and texts of laws and Special directives.

30
Total Marks : 200
Online Prelims TEST - 19 (SUBJECT WISE)
( InsightsIAS Mock Test Series for UPSC Preliminary Exam 2020 )

The legislature of a state may adopt any one or more of the languages in use in the state or Hindi as
the official language of that state. Until that is done, English is to continue as official language of
that state.

The Parliament enacted the Official Language Act in 1963. The act provides for the continued use of
English (even after 1965), in addition to Hindi, for all official purposes of the Union and also for the
transaction of business in Parliament. Notably, this act enables the use of English indefinitely
(without any time-limit). Further, this act was amended in 1967 to make the use of English, in
addition to Hindi, compulsory in certain cases.

48 Which of the following provisions of 73rd Constitutional Amendment Act (1992) or the Part IX of the
Constitution is/are the compulsory provisions/features?
1. Direct elections to all seats in panchayats at the village, intermediate and district levels.
2. Organisation of Gram Sabha in a village or group of villages.
3. Authorizing Panchayats to levy, collect and appropriate taxes, duties, tolls and fees

Select the correct answer using the code given below.


A. 1 and 2 only
B. 3 only
C. 2 and 3 only
D. 1, 2 and 3

Correct Answer : A

Answer Justification :

Compulsory Provisions of the 73rd Constitutional Amendment Act (1992) or the Part IX of
the Constitution:
1. Organisation of Gram Sabha in a village or group of villages. Hence, statement 2 is
correct.
2. Establishment of panchayats at the village, intermediate and district levels. Hence, statement 1
is correct.
3. Direct elections to all seats in panchayats at the village, intermediate and district levels.
4. Indirect elections to the post of chairperson of panchayats at the intermediate and district levels.
5. 21 years to be the minimum age for contesting elections to panchayats.
6. Reservation of seats (both members and chairpersons) for SCs and STs in panchayats at all the
three levels.
7. Reservation of one-third seats (both members and chairpersons) for women in panchayats at all
the three levels.
8. Fixing tenure of five years for panchayats at all levels and holding fresh elections within six
months in the event of supersession of any panchayat.
9. Establishment of a State Election Commission for conducting elections to the panchayats.
10. Constitution of a State Finance Commission after every five years to review the financial
position of the panchayats.

Voluntary Provisions:

31
Total Marks : 200
Online Prelims TEST - 19 (SUBJECT WISE)
( InsightsIAS Mock Test Series for UPSC Preliminary Exam 2020 )

1. Giving representation to members of the Parliament (both the Houses) and the state legislature
(both the Houses) in the panchayats at different levels falling within their constituencies.

2. Providing reservation of seats (both members and chairpersons) for backward classes in
panchayats at any level.

3. Granting powers and authority to the panchayats to enable them to function as institutions of
self-government (in brief, making them autonomous bodies).

4. Devolution of powers and responsibilities upon panchayats to prepare plans for economic
development and social justice; and to perform some or all of the 29 functions listed in the Eleventh
Schedule of the Constitution.

5. Granting financial powers to the pachayats, that is, authorizing them to levy, collect and
appropriate taxes, duties, tolls and fees. Hence, statement 3 is incorrect.

49 Which of the following is/are features or provisions of the “Provisions of the Panchayats (Extension to
the Scheduled Areas) Act”, 1996?
1. Planning and management of minor water bodies in the Scheduled Areas shall be entrusted to
Panchayats at the appropriate level.
2. A state legislation on the Panchayats in the Scheduled Areas shall be in consonance with the
customary law, social and religious practices and traditional management practices of community
resources.

Select the correct answer using the code given below.


A. 1 only
B. 2 only
C. Both 1 and 2
D. Neither 1 nor 2

Correct Answer : C

Answer Justification :

Both statements given above are correct.

The provisions of Part IX of the constitution relating to the Panchayats are not applicable to the
Fifth Schedule areas. However, the Parliament may extend these provisions to such areas, subject
to such exceptions and modifications as it may specify. Under this provision, the Parliament has
enacted the “Provisions of the Panchayats (Extension to the Scheduled Areas) Act”, 1996, popularly
known as the PESA Act or the Extension Act.

Some of the features (or the provisions) of the PESA Act are as follows:

A state legislation on the Panchayats in the Scheduled Areas shall be in consonance with the
customary law, social and religious practices and traditional management practices of community
resources.

32
Total Marks : 200
Online Prelims TEST - 19 (SUBJECT WISE)
( InsightsIAS Mock Test Series for UPSC Preliminary Exam 2020 )

Planning and management of minor water bodies in the Scheduled Areas shall be entrusted to
Panchayats at the appropriate level.

50 Which of the following is/are the features of the Family Courts Act, 1984?
1. It makes it obligatory on the State Governments to set up a Family Court in every city or town with
a population exceeding one million.
2. It provides for the establishment of Family Courts by the State Governments in consultation with
the High Courts.

Select the correct answer using the code given below:


A. 1 only
B. 2 only
C. Both 1 and 2
D. Neither 1 nor 2

Correct Answer : C

Answer Justification :

Both statements given above are correct.

The Family Courts Act, 1984 was enacted to provide for the establishment of Family Courts with a
view to promote conciliation and secure speedy settlement of disputes relating to marriage and
family affairs.

Family Courts Act, 1984 provides for the establishment of Family Courts by the State
Governments in consultation with the High Courts.

Family Courts Act, 1984 makes it obligatory on the State Governments to set up a Family
Court in every city or town with a population exceeding one million.

51 With reference to Attorney General of India, consider the following statements:


1. He has also the right to speak and take part in proceedings of both the houses of parliament
including joint sittings
2. He is not part of Union Executive.

Which of the statements given above is/are correct?


A. 1 only
B. 2 only
C. Both 1 and 2
D. Neither 1 nor 2

Correct Answer : A

Answer Justification :

33
Total Marks : 200
Online Prelims TEST - 19 (SUBJECT WISE)
( InsightsIAS Mock Test Series for UPSC Preliminary Exam 2020 )

The Attorney General (AG) is appointed by the president. He must be a person who is qualified to be
appointed a judge of the Supreme Court. In other words, he must be a citizen of India and he must
have been a judge of some high court for five years or an advocate of some high court for ten years
or an eminent jurist, in the opinion of the president. He is part of Union Executive. Hence,
statement 2 is incorrect.

In the performance of his official duties, the Attorney General has the right of audience in all courts
in the territory of India. Further, he has the right to speak and to take part in the proceedings of
both the Houses of Parliament or their joint sitting and any committee of the Parliament of which he
may be named a member, but without a right to vote. Hence, statement 1 is correct.

52 Consider the following statements regarding the Election Commission of India


1. Election Commission of India is a permanent body entrusted for the conduct of free and fair
elections.
2. The Election Commission consists of five members.
3. If the chief election commissioner and other election commissioners differ in opinion on any matter,
such matter shall be decided according to the opinion of the majority.

Which of the statements given above is/are correct?


A. 2 only
B. 1 and 3 only
C. 2 and 3 only
D. 1, 2 and 3

Correct Answer : B

Answer Justification :

The Election Commission is a permanent and an independent body established by the Constitution
of India directly to ensure free and fair elections in the country. Article 324 of the Constitution
provides that the power of superintendence, direction and control of elections to parliament, state
legislatures, the office of president of India and the office of vicepresident of India shall be vested in
the election commission. Hence, statement 1 is correct.

The Election Commission has been functioning as a multi-member body consisting of three election
commissioners. Hence, statement 2 is incorrect.

The chief election commissioner and the two other election commissioners have equal powers and
receive equal salary, allowances and other perquisites, which are similar to those of a judge of the
Supreme Court.3 In case of difference of opinion amongst the Chief election commissioner and/or
two other election commissioners, the matter is decided by the Commission by majority. Hence,
statement 3 is correct.

53 With reference to NITI Aayog, consider the following statements


1. It is neither a constitutional nor a statutory body.
2. NITI Aayog plays important role in designing strategic and long term policies and programmes for

34
Total Marks : 200
Online Prelims TEST - 19 (SUBJECT WISE)
( InsightsIAS Mock Test Series for UPSC Preliminary Exam 2020 )

the Government.

Which of the statements given above is/are correct?


A. 1 only
B. 2 only
C. Both 1 and 2
D. Neither 1 nor 2

Correct Answer : C

Answer Justification :

The NITI Aayog is a policy think tank of the Government of India, established with the aim to
achieve sustainable development goals with cooperative federalism by fostering the involvement of
State Governments of India in the economic policy-making process using a bottom-up approach.

NITI AYOG has never been passed as an act of Parliament nor it has been mentioned anywhere in
the Constitution thus it’s definitely neither statutory nor a constitutional body. Hence, statement 1
is correct.

NITI Aayog plays important role in designing strategic and long term policies and programmes for
the Government. It also provides relevant technical advice to the Centre and States. Hence,
statement 2 is correct.

54 With reference to National Human Rights Commission (NHRC), consider the following statements
1. NHRC is an independent statutory body
2. NHRC investigates grievances regarding the violation of human rights either suo moto or after
receiving a petition.
3. The chairman of NHRC should be a retired chief justice of India.

Which of the statements given above is/are correct?


A. 2 only
B. 1 and 3 only
C. 2 and 3 only
D. 1, 2 and 3

Correct Answer : D

Answer Justification :

The National Human Rights Commission of India is a Statutory public body constituted on 12
October 1993 under the Protection of Human Rights Ordinance of 28 September 1993. It was given
a statutory basis by the Protection of Human Rights Act, 1993. Hence, statement 1 is correct.

NHRC comprises of a chairman and four members. The chairman should be a retired chief justice of
India. Hence, statement 3 is correct.

35
Total Marks : 200
Online Prelims TEST - 19 (SUBJECT WISE)
( InsightsIAS Mock Test Series for UPSC Preliminary Exam 2020 )

NHRC investigates grievances regarding the violation of human rights either suo moto or after
receiving a petition. It has the power to interfere in any judicial proceedings involving any
allegation of violation of human rights. It can visit any jail or any other institution under the control
of the State Government to see the living conditions of the inmates and to make recommendations
thereon. Hence, statement 2 is correct.

55 With reference to the Central Vigilance Commissioner, consider the following statements:
1. It is an investigating agency.
2. The Central Vigilance Commissioner holds the office for 5 years
3. It submits its report to the president of India.

Which of the statements given above is/are correct?


A. 3 only
B. 1 and 3 only
C. 2 and 3 only
D. 1, 2 and 3

Correct Answer : A

Answer Justification :

Central Vigilance Commission is an apex Indian governmental body created in 1964 to address
governmental corruption. In 2003, the Parliament enacted a law conferring statutory status on the
CVC. It is not an investigating agency. The CVC either gets the investigation done through the CBI
or through chief vigilance officers (CVO) in government offices. Hence, statement 1 is incorrect.

The Central Vigilance Commissioner is to be appointed by the President of India. He holds the office
for 4 years. He can be removed or suspended from the office by the President on the ground of
misbehavior but only after the Supreme Court has held an inquiry into his case and recommended
action against him. Hence, statement 2 is incorrect.

It submits its report to the President of India. Hence, statement 3 is correct.

56 The Chief Information Commissioner and Information Commissioners are appointed by the President
on the recommendation of a committee which consisting of:
1. The Prime Minister
2. The Leader of Opposition in the Lok Sabha.
3. A Union Cabinet Minister to be nominated by the Prime Minister.
4. Chief justice of India.

Which of the statements given above is/are correct?


A. 1 and 2 only
B. 2 and 3 only
C. 1, 2 and 3 only
D. 1, 2, 3 and 4

36
Total Marks : 200
Online Prelims TEST - 19 (SUBJECT WISE)
( InsightsIAS Mock Test Series for UPSC Preliminary Exam 2020 )

Correct Answer : C

Answer Justification :

Central Information Commission includes 1 Chief Information Commissioner (CIC) and not more
than 10 Information Commissioners (IC) who are appointed by the President of India on the
recommendations of a Committee.

The Chief Information Commissioner and Information Commissioners are appointed by the
President on the recommendation of a committee consisting of—

The Prime Minister, who shall be the Chairperson of the committee. Hence, statement 1 is
correct.

The Leader of Opposition in the Lok Sabha. Hence, statement 2 is correct.

A Union Cabinet Minister to be nominated by the Prime Minister. Hence, statement 3 is


correct.

57 With reference to the Union Public Service Commission (UPSC), consider the following statements:
1. UPSC is a regulatory body
2. It conducts examinations for appointments to the services of the union, which includes all India
services, central services and public services of the union territories.
3. The Chairman and the members of the commission hold office for the period of the 6 years or until
the age of 65 years.

Which of the statements given above is/are correct?


A. 2 only
B. 1 and 3 only
C. 2 and 3 only
D. 1, 2 and 3

Correct Answer : C

Answer Justification :

The Union Public Service Commission is India's premier central recruiting agency. It is responsible
for appointments to and examinations for All India services and group A & group B of Central
services. While Department of Personnel and Training is the central personnel agency in India.
Hence, statement 2 is correct.

37
Total Marks : 200
Online Prelims TEST - 19 (SUBJECT WISE)
( InsightsIAS Mock Test Series for UPSC Preliminary Exam 2020 )

UPSC is a constitutional body, it has been established under Article 315 of the Constitution of India
and consists of a Chairman and ten Members, who are appointed and removed by President.
Hence, statement 1 is incorrect.

The chairman and members of the commission hold office for a term of six years or until they attain
the age of 65 years, whichever is earlier. The members can resign in between the term by
addressing their resignation to the president. They can also be removed by the president following
the procedure provided in the constitution. Hence, statement 3 is correct.

58 Consider the following statements


1. Joint State Public Service Commission is a constitutional body.
2. The number of members of a JSPSC and their conditions of service are determined by the
parliament.

Which of the statements given above is/are correct?


A. 1 only
B. 2 only
C. Both 1 and 2
D. Neither 1 nor 2

Correct Answer : B

Answer Justification :

The Constitution makes a provision for the establishment of a Joint State Public Service Commission
(JSPSC) for two or more states. While the UPSC and the SPSC are created directly by the
Constitution, a JSPSC can be created by an act of Parliament on the request of the state legislatures
concerned. Thus, a JSPSC is a statutory and not a constitutional body. Hence, statement 1 is
incorrect.

The number of members of a JSPSC and their conditions of service are determined by the president.
A JSPSC presents its annual performance report to each of the concerned state governors. Each
governor places the report before the state legislature. Hence, statement 2 is correct.

59 With reference to Finance Commission, consider the following statements.


1. It is constituted by the Finance minister of India every fifth year
2. The commission submits its report to the president.

Which of the statements given above is/are correct?


A. 1 only
B. 2 only
C. Both 1 and 2
D. Neither 1 nor 2

Correct Answer : B

38
Total Marks : 200
Online Prelims TEST - 19 (SUBJECT WISE)
( InsightsIAS Mock Test Series for UPSC Preliminary Exam 2020 )

Answer Justification :

Article 280 of the Constitution of India provides for a Finance Commission as a quasi-judicial body.
It is constituted by the president of India every fifth year or at such earlier time as he considers
necessary. Hence, statement 1 is incorrect.

The commission submits its report to the president. He lays it before both the Houses of Parliament
along with an explanatory memorandum as to the action taken on its recommendations. Hence,
statement 2 is correct.

60 With reference to National Commission for Scheduled Castes, consider the following statements
1. It is a statutory body established with a view to provide safeguards against the exploitation of
Scheduled Castes.
2. The Commission consists of a chairperson, a vice-chairperson and three other members.

Which of the statements given above is/are correct?


A. 1 only
B. 2 only
C. Both 1 and 2
D. Neither 1 nor 2

Correct Answer : B

Answer Justification :

National Commission for Scheduled Caste is a constitutional body under article 338 appointed by
the President. It has been established with a view to provide safeguards against the exploitation of
Scheduled Caste. In addition, to promote and protect their social, educational, economic and
cultural interests, special provisions were made in the Constitution. Hence, statement 1 is
incorrect.

The Commission consists of a chairperson, a vice-chairperson and three other members. They are
appointed by the President by warrant under his hand and seal. Hence, statement 2 is correct.

61 With reference to the Cabinet Secretariat, consider the following statements:


1. It functions directly under the Ministry of Parliamentary Affairs.
2. The Cabinet Secretary is the ex-officio Chairman of the Civil Services Board.
3. It is responsible for facilitating smooth transaction of business in Ministries/ Departments.

Which of the statements given above is/are correct?


A. 1 only
B. 2 and 3 only
C. 1 and 3 only
D. 3 only

39
Total Marks : 200
Online Prelims TEST - 19 (SUBJECT WISE)
( InsightsIAS Mock Test Series for UPSC Preliminary Exam 2020 )

Correct Answer : B

Answer Justification :

The Cabinet Secretariat functions directly under the Prime Minister. Hence, statement 1 is
incorrect.

The administrative head of the Secretariat is the Cabinet Secretary who is also the ex-officio
Chairman of the Civil Services Board. Hence, statement 2 is correct.

The business allocated to Cabinet Secretariat under Government of India (Allocation of Business)
Rules, 1961 includes (i) Secretarial assistance to the Cabinet and Cabinet Committees; and (ii)
Rules of Business.

The Cabinet Secretariat is responsible for the administration of the Government of India
(Transaction of Business) Rules, 1961 and Government of India (Allocation of Business)
Rules, 1961 facilitating smooth transaction of business in Ministries/ Departments. Hence,
statement 3 is correct.

The Secretariat assists in decision-making in Government by ensuring Inter-Ministerial


coordination, ironing out differences amongst Ministries/Departments and evolving consensus
through the instrumentality of the standing/adhoc Committees of Secretaries. Management of major
crisis situations in the country and coordinating activities of various ministries in such a situation is
also one of the functions of the Cabinet Secretariat.

https://cabsec.gov.in/about/functions/

62 Consider the following statements:


1. The Project Monitoring Group (PMG) functions under the Prime Minister’s Office (PMO).
2. PMG in association helps foreign investors intending to make large investments in India by
facilitating approvals/clearances.

Which of the statements given above is/are correct?


A. 1 only
B. 2 only
C. Both 1 and 2
D. Neither 1 nor 2

Correct Answer : C

Answer Justification :

Both statements given above are correct.

The Project Monitoring Group (PMG) was set up in 2013 under Cabinet Secretariat. It is an
institutional mechanism for resolving a variety of issues including fast tracking the approvals for
setting up and expeditious commissioning of large Public, Private and Public–Private Partnership

40
Total Marks : 200
Online Prelims TEST - 19 (SUBJECT WISE)
( InsightsIAS Mock Test Series for UPSC Preliminary Exam 2020 )

(PPP) Projects. PMG is now functioning under Prime Minister’s Office (PMO) since
14.09.2015.

PMG in association with ‘Invest India’, the agency dedicated to promotion of foreign
Investment in India, also helps foreign investors intending to make large investments in
India by facilitating approvals/clearances and providing them necessary support during
implementation of projects.

https://www.pmindia.gov.in/en/role-of-project-monitoring-group-pmg/

63 Which of the following factors is/are considered by the Commission for Agricultural Costs & Prices
(CACP) while recommending the minimum support price (MSP)?
1. Cost of production
2. Inter-crop price parity
3. Likely implications of MSP on consumers of that product.

Select the correct answer using the code given below:


A. 1 and 2 only
B. 2 and 3 only
C. 1 and 3 only
D. 1, 2 and 3

Correct Answer : D

Answer Justification :

All the factors given above are considered.

While recommending price policy of various commodities under its mandate, the Commission keeps
in mind the various Terms of Reference (ToR) given to CACP in 2009. Accordingly, it analyzes

1) demand and supply;


2) cost of production;
3) price trends in the market, both domestic and international;
4) inter-crop price parity;
5) terms of trade between agriculture and non-agriculture;
6) a minimum of 50 percent as the margin over cost of production; and
7) likely implications of MSP on consumers of that product.

It may be noted that cost of production is an important factor that goes as an input in determination
of MSP, but it is certainly not the only factor that determines MSP.

64 With reference to the Commission for Agricultural Costs and Prices (CACP), consider the following
statements:
1. It is an autonomous body under Ministry of Agriculture and Farmers Welfare.
2. It do not recommend minimum support price for any commercial crop.

41
Total Marks : 200
Online Prelims TEST - 19 (SUBJECT WISE)
( InsightsIAS Mock Test Series for UPSC Preliminary Exam 2020 )

Which of the following statements given above is/are correct?


A. 1 only
B. 2 only
C. Both 1 and 2
D. Neither 1 nor 2

Correct Answer : D

Answer Justification :

Both the statements given above are incorrect.

The Commission for Agricultural Costs & Prices (CACP) is an attached office of the Ministry of
Agriculture and Farmers Welfare, Government of India. It came into existence in January 1965.

As of now, CACP recommends MSPs of 23 commodities, which comprise 7 cereals (paddy, wheat,
maize, sorghum, pearl millet, barley and ragi), 5 pulses (gram, tur, moong, urad, lentil), 7 oilseeds
(groundnut, rapeseed-mustard, soyabean, seasmum, sunflower, safflower, nigerseed), and 4
commercial crops (copra, sugarcane, cotton and raw jute).

https://cacp.dacnet.nic.in/content.aspx?pid=32

65 One Hundred and Third (103rd) constitution amendment relates to:

A. Establishment of Goods and Service Tax Council (GSTC)


B. Reservation to the Economically Weaker section in employment and Educational institutions.

C. Establishment of National Commission for Backward Classes.


D. Exchange of land enclaves between India and Bangladesh.

Correct Answer : B

Answer Justification :

The One Hundred and Third Amendment of the Constitution of India, officially known as
the Constitution (One Hundred and Third Amendment) Act, 2019, introduces 10%
reservation for Economically Weaker Sections (EWS) of society for admission to Central
Government-run educational institutions and private educationally institutions (except for
minority educational institutions), and for employment in Cent96383333ral Government jobs.

Hence, option (b) is correct.

66 With reference to the National Sports Development Fund (NSDF), consider the following statements:
1. It was established under the Charitable Endowments Act 1890.
2. Donor can suggest execution of specific projects while making donations to the Fund.

42
Total Marks : 200
Online Prelims TEST - 19 (SUBJECT WISE)
( InsightsIAS Mock Test Series for UPSC Preliminary Exam 2020 )

3. Contribution to the Fund is exempt from Income Tax.

Which of the statements given above is/are correct?


A. 1 and 2 only
B. 2 and 3 only
C. 1 and 3 only
D. 1, 2 and 3

Correct Answer : D

Answer Justification :

All the statements given above are correct.

National Sports Development Fund (NSDF) was established in 1998 under the Charitable
Endowments Act 1890. Purpose of creation is to impart momentum and flexibility to assisting the
cause of sports; The Fund helps sportspersons excel by providing them opportunities to train under
coaches of international repute with technical, scientific and psychological support and giving them
exposure to international competitions; The Fund also provides financial assistance for development
of infrastructure and other activities for promotion of sports; Role of the Fund is supplementary to
the overall policy and activities of the Department of Sports in achieving excellence in sports.

ADVANTAGES OF CONTRIBUTING TO NSDF

Donor can suggest execution of specific projects while making donations to the
Fund.

Fulfilment of CSR obligations of companies under the Companies Act, 2013. Sports activities
like training to promote sports and creation and maintenance of sports infrastructure are
covered under Corporate Social Responsibility (CSR).

Contribution to the Fund is exempt from Income Tax under Section 80 (G) of the
Income Tax Act.

Due credit is given to donors in the Ministry’s publications, media releases, etc.

Goodwill for donors.

Meetings of donors with dignitaries.

Consideration for Khel Protsahan Puraskar which is given away by the President of India
every year.

43
Total Marks : 200
Online Prelims TEST - 19 (SUBJECT WISE)
( InsightsIAS Mock Test Series for UPSC Preliminary Exam 2020 )

Athlete appearances for donors who contribute at least one crore a year.

https://yas.nic.in/sports/national-sports-development-fund-0

67 With reference to the National Backward Classes Finance & Development Corporation (NBCFDC),
consider the following statements:
1. It is incorporated as ‘not for profit’ company.
2. It facilitates skill development of poor persons belonging to De-notified Nomadic and Semi-Nomadic
tribe as well.
3. It provides micro financing through State Channelizing Agencies (SCAs)/ Self Help Groups (SHGs).

Which of the statements given above is/are correct?


A. 1 and 2 only
B. 2 and 3 only
C. 1 and 3 only
D. 1, 2 and 3

Correct Answer : D

Answer Justification :

All the statements given above are correct.

National Backward Classes Finance & Development Corporation (NBCFDC) is a Govt. of


India Undertaking under the aegis of Ministry of Social Justice and Empowerment. NBCFDC was
incorporated under Section 25 of the Companies Act 1956 on 13th January 1992 (now
section 8 of Companies Act 2013) as a Company not for profit with an objective to promote
economic and developmental activities for the benefit of Other Backward Classes (OBCs) and to
assist the poorer section of these classes in skill development and self-employment ventures.
NBCFDC provides financial assistance through State Channelizing Agencies (SCAs) nominated by
the State Governments/UTs and Banks (RRBs & PSBs). NBCFDC also provides Micro
Financing through SCAs/ Self Help Groups (SHGs). The Corporation can assist a wide range of
income generating activities to assist the poorer section of these classes in self-employment
ventures under following broad sectors:

Agriculture and Allied Activities

Small Business/Artisan and Traditional Occupation

Transport and Service Sector etc.

Technical, Vocational and Professional Trades/Courses

44
Total Marks : 200
Online Prelims TEST - 19 (SUBJECT WISE)
( InsightsIAS Mock Test Series for UPSC Preliminary Exam 2020 )

The Corporation additionally facilitates skill development of poor persons belonging to


OBCs, persons of Economically Backward Classes (EBCs), De-notified Nomadic and Semi-
Nomadic Tribe, Sr. Citizen, Beggars and Transgender.

https://nbcfdc.gov.in/corporate/en

68 Consider the following statements:


1. The Agricultural and Processed Food Products Export Development Authority (APEDA) is a non-
constitutional and non-statutory body.
2. The Tribal Cooperative Marketing Development Federation of India (TRIFED) came is a national-
level apex organization functioning under the administrative control of Ministry of Tribal Affairs.

Which of the statements given above is/are NOT correct?


A. 1 only
B. 2 only
C. Both 1 and 2
D. Neither 1 nor 2

Correct Answer : A

Answer Justification :

The Agricultural and Processed Food Products Export Development Authority (APEDA) was
established by the Government of India under the Agricultural and Processed Food Products Export
Development Authority Act passed by the Parliament in December, 1985. The Authority replaced
the Processed Food Export Promotion Council (PFEPC). Hence, statement 1 is incorrect.

The Tribal Cooperative Marketing Development Federation of India (TRIFED) came into existence
in 1987. It is a national-level apex organization functioning under the administrative
control of Ministry of Tribal Affairs, Govt. of India. Hence, statement 2 is correct.

69 Consider the following statements:


1. A community declared as scheduled tribe in one state need not to be so in other states.
2. Parliament may by law include or exclude any tribe from the list of Scheduled Tribes.

Which of the statements given above is/are correct?


A. 1 only
B. 2 only
C. Both 1 and 2
D. Neither 1 nor 2

Correct Answer : C

Answer Justification :

45
Total Marks : 200
Online Prelims TEST - 19 (SUBJECT WISE)
( InsightsIAS Mock Test Series for UPSC Preliminary Exam 2020 )

Both statements given above are correct.

Under article 342(1), the President may, with respect to any State or Union territory, and
where it is a state, after consultation with the Governor there of by public notification, specify
the tribes or tribal communities or parts of or groups within tribes or tribal communities which
shall, for the purposes of this constitution, is deemed to be scheduled tribes in relation to that state
or Union Territory, as the case may be.

Parliament may by law include in or exclude from the list of Scheduled Tribes specified in
a notification issued under article 342 (2) any tribe or tribal community or part of or
group within any tribe or tribal community, but save as aforesaid a notification issued under
the said clause shall not be varied by any subsequent notification.

The list of Scheduled Tribes is State/UT specific and a community declared as a Scheduled
Tribe in a State need not be so in another State. The inclusion of a community as a Scheduled
Tribe is an ongoing process.

http://vikaspedia.in/social-welfare/scheduled-tribes-welfare/ministry-of-tribal-welfare

70 Consider the following statements:


1. Central Ground Water Authority has been constituted under the Water Act, 1974.
2. Central Ground Water Board (CGWB is the national apex agency entrusted with the responsibilities
of providing scientific inputs for management, exploration, monitoring, assessment, augmentation
and regulation of ground water resources of the country.
3. Dam Rehabilitation and Improvement Project (DRIP) is a project for the repair and rehabilitation of
dam projects across the seven states of India with the financial assistance from Asian Development
Bank (ADB).

Which of the statements given above is/are correct?


A. 2 only
B. 1 and 3 only
C. 1 and 2 only
D. 2 and 3 only

Correct Answer : A

Answer Justification :

Central Ground Water Authority has been constituted under Section 3 (3) of the
Environment (Protection) Act, 1986 to regulate and control development and management of
ground water resources in the country. Hence, statement 1 is incorrect.

Central Ground Water Board (CGWB), a subordinate office of the Ministry of Water Resources,
Government of India, is the National Apex Agency entrusted with the responsibilities of
providing scientific inputs for management, exploration, monitoring, assessment,
augmentation and regulation of ground water resources of the country. Central Ground
Water Board was established in 1970 by renaming the Exploratory Tube wells Organization under

46
Total Marks : 200
Online Prelims TEST - 19 (SUBJECT WISE)
( InsightsIAS Mock Test Series for UPSC Preliminary Exam 2020 )

the Ministry of Agriculture, Government of India. It was merged with the Ground Water Wing of the
Geological Survey of India during 1972. Hence, statement 2 is correct.

Govt. of India has taken up the Dam Rehabilitation and Improvement Project (DRIP) with
the financial assistance of the World Bank for the repair and rehabilitation of initially
about 225 dam projects across the seven states of India, namely Jharkhand (DVC),
Karnataka, Kerala, Madhya Pradesh, Odisha, Tamil Nadu, and Uttarakhand (UJVNL).
Hence, statement 3 is incorrect.

71 Consider the following statements regarding Vembanad Lake


1. It is designated as Ramsar Site
2. It is the longest lake in India
3. Pamba and Periyar rivers drains into it

Which of the statements given above is/are correct?


A. 2 and 3 only
B. 3 only
C. 1 only
D. 1, 2 and 3

Correct Answer : D

Answer Justification :

All the statements given above are correct.

Vembanad (Vembanad Kayal or Vembanad Kol) is the longest lake in India, and the largest
lake in the state of Kerala. [Mangrove with area 2114 sq. Km is the largest Ramasar site in India]

Spanning several districts in the state of Kerala, it is known as Vembanadu Lake in Alappuzha,
Punnamada Lake in Kuttanad and Kochi Lake in Kochi.

Several groups of small islands including Vypin, Mulavukad, Vallarpadam, Willingdon Island are
located in the Kochi Lake portion. Kochi Port is built around the Willingdon Island and the
Vallarpadam Island.

It is designated as Ramsar Site

Pamba and Periyar rivers drains into it.

https://www.thehindu.com/news/national/kerala/high-coliform-content-in-vembanad-lake/article3006
3725.ece

72 Consider the following statements regarding Chenchus Tribe


1. They are endemic to Central India
2. They help in the conservation of the Tigers
3. They have been categorized as Particularly Vulnerable Tribal Groups
47
Total Marks : 200
Online Prelims TEST - 19 (SUBJECT WISE)
( InsightsIAS Mock Test Series for UPSC Preliminary Exam 2020 )

Which of the statements given above is/are correct?


A. 1 only
B. 2 and 3 only
C. 3 only
D. 1, 2 and 3

Correct Answer : B

Answer Justification :

The Chenchus are Adivasi, a designated Scheduled Tribe in the Indian states of Andhra
Pradesh, Telangana, Karnataka and Odisha. Hence Statement 1 is incorrect.

They are an aboriginal tribe whose traditional way of life has been based on hunting and gathering.

They help to conserve the big cats in Nallamala Forest. Hence Statement 2 is correct.

They have been categorized as Particularly Vulnerable Tribal Groups. Hence Statement 3 is
correct.

https://www.thehindu.com/news/national/andhra-pradesh/hidden-chenchus-and-crouching-tigers-of-
nallamala/article28763206.ece

http://vikaspedia.in/social-welfare/scheduled-tribes-welfare/primitive-vulnerable-tribal-groups

73 Under which of the following acts an individual can be designate as Fugitive Economic Offender
⌰〰〰
1. Reserve Bank of India Act, 1934
2. Prevention of Money Laundering Act, 2002
3. Central Excise Act, 1944

Select the correct answer using the code given below


A. 2 only
B. 1 and 2 only
C. 1, 2 and 3
D. None

Correct Answer : C

Answer Justification :

A fugitive economic offender is any individual against whom a warrant for arrest in relation to a
scheduled offence has been issued by any court in India and who has either left India to avoid
criminal prosecution, or who, being abroad, refuses to return to India to face criminal prosecution

The list of offences that can qualify an individual to be designated an economic offender,

48
Total Marks : 200
Online Prelims TEST - 19 (SUBJECT WISE)
( InsightsIAS Mock Test Series for UPSC Preliminary Exam 2020 )

enumerated includes offences under several Acts such as the Negotiable Instruments Act, 1881; the
Reserve Bank of India Act, 1934; the Central Excise Act, 1944; the Customs Act, 1962; the
Prohibition of Benami Property Transactions Act, 1988; the Prevention of Money Laundering Act,
2002; and the Indian Penal Code.

https://www.thehindu.com/opinion/op-ed/explaining-the-fugitive-economic-offenders-ordinance/artic
le24454186.ece

74 Consider the following statements regarding 4th National Tiger Estimation


1. Chhattisgarh and Mizoram saw a decline in tiger population and all other States saw a positive
increase.
2. Bandipur Tiger Reserve in Karnataka recorded the highest number of tigers.
3. Madhya Pradesh has highest number of tigers at 996.

Which of the statements given above is/are not correct?


A. 2 and 3 only
B. 1 and 3 only
C. 1 only
D. 1 and 2 only

Correct Answer : A

Answer Justification :

4th National Tiger Estimation (Tiger-census)

India has 2,967 tigers.

Madhya Pradesh saw the highest number at 526, closely followed by


Karnataka (524) and Uttarakhand (442). Hence statement 3 is incorrect.

Chhattisgarh and Mizoram saw a decline in tiger population and all other States saw a positive
increase. Hence statement 1 is correct.

While the Pench Tiger Reserve in Madhya Pradesh recorded the highest number of tigers, the
Sathyamangalam Tiger Reserve in Tamil Nadu registered the “maximum improvement” since 2014.
Hence statement 2 is incorrect.

https://www.thehindu.com/news/national/nearly-3000-tigers-in-india-finds-census/article28744392.e
ce

75 Consider the following pairs


List I List II
Kaundinya Wildlife Sanctuary Kerala
Bhadra Wildlife Sanctuary Karnataka
Parambikulam Tiger reserve Tamil Nadu

49
Total Marks : 200
Online Prelims TEST - 19 (SUBJECT WISE)
( InsightsIAS Mock Test Series for UPSC Preliminary Exam 2020 )

Which of the statements given above is/are correct?


A. 2 and 3 only
B. 3 only
C. 1 and 2 only
D. 2 only

Correct Answer : D

Answer Justification :

Kaundinya Wildlife Sanctuary is a wildlife sanctuary and an elephant reserve situated in Andhra
Pradesh, India.

Bhadra Wildlife Sanctuary is a protected area and a tiger reserve as part of Project Tiger, located
23 km south of Bhadravathi town and 38 km northwest of Chikkamagaluru town in Karnataka
state, India

Parambikulam Tiger Reserve, which also includes the erstwhile Parambikulam Wildlife
Sanctuary, is a 391 square kilometres protected area in Chittur taluk in Palakkad district of Kerala
state, South India.

76 Women Transforming India (WTI) Awards 2019, sometime seen in the news, is flagship initiative of

A. Ministry of Women and Child Development


B. NITI Ayog
C. Ministry of Commerce and Industry
D. None of the above

Correct Answer : B

Answer Justification :

Women Entrepreneurship Platform, a flagship initiative of NITI Aayog, has announced the fourth
edition of Women Transforming India (WTI) Awards, which is being organised in collaboration with
the United Nations.

https://niti.gov.in/women-transforming-india

77 Consider the following statements regarding Geographical Indications (GIs) Tag


1. GIs have been defined under WTO Agreement on Trade-Related Aspects of Intellectual Property
Rights (TRIPS) Agreement
2. Civil and Criminal proceedings can be initiated against those using the logo in authorized manner
3. The registration of GI Tag is valid for 10 years after which it needs to be renewed.

Which of the statements given above is/are correct?

50
Total Marks : 200
Online Prelims TEST - 19 (SUBJECT WISE)
( InsightsIAS Mock Test Series for UPSC Preliminary Exam 2020 )

A. 1 and 2 only
B. 1 only
C. 2 only
D. 1, 2 and 3

Correct Answer : D

Answer Justification :

All the statements given above are correct.

A geographical indication (GI) is a name or sign used on certain products which corresponds to a
specific geographical location or origin (e.g., a town, region, or country). India, as a member of
the World Trade Organization (WTO), enacted the Geographical Indications of Goods
(Registration and Protection) Act, 1999 has come into force with effect from 15 September
2003.

The GI tag ensures that none other than those registered as authorised users (or at least those
residing inside the geographic territory) are allowed to use the popular product name. Darjeeling
tea became the first GI tagged product in India, in 2004–2005

Civil and Criminal proceedings can be initiated against those using the logo in authorized
manner

The registration of GI Tag is valid for 10 years after which it needs to be renewed.

https://www.thehindu.com/news/national/tamil-nadu/dindigul-locks-to-kandangi-saris-would-gi-tags-r
evive-an-industry/article29311542.ece

78 Which of the following species is/are extinct in India?


1. Cheetah
2. Sumatran Rhino
3. The Pink headed duck

Select the correct answer using the code given below


A. 2 and 3 only
B. 1 and 3 only
C. 1 only
D. 1, 2 and 3

Correct Answer : D

Answer Justification :

Among mammals, the cheetah (Acionyxjubatus) and the Sumatran rhinoceros


(Dicerorhinussumatrensisi) are considered extinct in India.

51
Total Marks : 200
Online Prelims TEST - 19 (SUBJECT WISE)
( InsightsIAS Mock Test Series for UPSC Preliminary Exam 2020 )

The pink-headed duck (Rhodonessacaryophyllaceai) is feared extinct since 1950 and the Himalayan
quail (Ophrysiasupercililios) was last reported in 1876.

79 Dolutegravir pills, sometime seen in the news, fights against which of the following disease or virus?

A. Nipah Virus
B. HIV Virus
C. Multi Drug Resistance TB
D. Ebola Virus

Correct Answer : B

Answer Justification :

Based on new evidence assessing benefits and risks, the World Health Organisation (WHO) has
recommended the use of the HIV drug dolutegravir (DTG) as the preferred first-line and second-line
treatment for all populations, including pregnant women and those of childbearing potential.

https://www.thehindu.com/news/national/who-recommends-use-of-innovative-hiv-drug/article287330
64.ece

80 Kambala, sometime seen in the news, is being held in?

A. Kerala
B. Tamil Nadu
C. Andhra Pradesh
D. Karnataka

Correct Answer : D

Answer Justification :

A kambala is an annual buffalo race held in the southwestern Indian state of Karnataka.
Traditionally, it is sponsored by local Tuluva landlords and households in the coastal districts of
Dakshina Kannada and Udupi, a region collectively known as Tulu Nadu.

81 Consider the following statements regarding Senescent cells


1. It is a type of stem cells.
2. It produce high amount of nucleotides that builds DNA

Which of the statements given above is/are correct?


A. 1 only
B. 2 only
C. Both 1 and 2

52
Total Marks : 200
Online Prelims TEST - 19 (SUBJECT WISE)
( InsightsIAS Mock Test Series for UPSC Preliminary Exam 2020 )

D. Neither 1 nor 2

Correct Answer : D

Answer Justification :

Both the statements are incorrect.

Senescent cells are the opposite of stem cells, they can never divide again. New research finds
that ageing, senescent cells stop producing nucleotides, the building block of DNA. When young
cells were prevented from producing nucleotides, the cells became senescent.

https://www.sciencedirect.com/topics/medicine-and-dentistry/cellular-senescence

82 Consider the following statements Antarctic Oscillation


1. It is the dominant pattern of seasonal tropospheric circulation variations south of 20S.
2. It has indirect effect on Indian climate system.

Which of the statements given above is/are correct?


A. 1 only
B. 2 only
C. Both 1 and 2
D. Neither 1 nor 2

Correct Answer : C

Answer Justification :

Both the statements given above are correct.

The Antarctic Oscillation (AAO) is the dominant pattern of non-seasonal tropospheric


circulation variations south of 20S, and it is characterized by pressure anomalies of one sign
centered in the Antarctic and anomalies of the opposite sign centered about 40-50S.

It does not act directly to influence Indian climate but affects the Indian Ocean Meridional Dipole
which in turn plays a role in our climatic conditions.

http://research.jisao.washington.edu/data/aao/

83 Which of the following agriculture practices is/are promoted by Zero Budget Natural Farming
(ZBNF)?
1. Soil Aeration
2. Intercropping
3. Topsoil Mulching
4. Vermicomposting

53
Total Marks : 200
Online Prelims TEST - 19 (SUBJECT WISE)
( InsightsIAS Mock Test Series for UPSC Preliminary Exam 2020 )

Select the correct answer using the code given below


A. 2 and 3 only
B. 3 and 4 only
C. 1, 2 and 3 only
D. 1, 2, 3 and 4

Correct Answer : C

Answer Justification :

The Zero Budget Natural Farming (ZBNF) method also promotes soil aeration, minimal watering,
intercropping, bunds and topsoil mulching and discourages intensive irrigation and deep ploughing.
Hence Statement 1, 2 and 3 are correct.

Zero Budget Natural Farming (ZBNF) is against vermicomposting, which is the mainstay of typical
organic farming, as it introduces the the most common composting worm, the European red wiggler
(Eisenia fetida) to Indian soils. Hence Statement 4 is incorrect.

84 Consider the following statements regarding Goods & Services Tax (GST) Council
1. It is a constitutional body.
2. GST Council is chaired by Prime Minister

Which of the statements given above is/are correct?


A. 1 only
B. 2 only
C. Both 1 and 2
D. Neither 1 nor 2

Correct Answer : A

Answer Justification :

It is a constitutional body (as per Article 279A of the amended Constitution) for making
recommendations to the Union and State Government on issues related to GST. Hence Statement
1 is correct.

The GST Council is chaired by the Union Finance Minister and other members are the Union State
Minister of Revenue or Finance and Ministers in-charge of Finance or Taxation of all the States.
Hence Statement 2 is incorrect.

85 Consider the following pairs


Military Exercise Participating countries
1. AUSIINDEX India and Australia
2. Varuna Russia and India
3. Yudh Abhyas India and Nepal

54
Total Marks : 200
Online Prelims TEST - 19 (SUBJECT WISE)
( InsightsIAS Mock Test Series for UPSC Preliminary Exam 2020 )

Which of the pairs given above is/are correct?


A. 1 and 3 only
B. 1 and 2 only
C. 1 only
D. None

Correct Answer : C

Answer Justification :

Yudh Abhyas is one of the largest running joint military training and defence corporation
endeavours between India and the US

Varuna naval exercise is an integral part of France–India strategic relationship in the 21st
century and consists of naval cooperation drills between the French Navy and the Indian Navy.

AUSINDEX, a joint maritime exercise between India and Australia

86 Consider the following statements regarding Bharat Stage VI Norms


1. India will move to Bharat Stage VI mass emission standards by April 2020
2. It sets limit for particle number in engines
3. Bharat Stage VI Norms is based on US Emission standards

Which of the statements given above is/are correct?


A. 1 only
B. 1 and 2 only
C. 2 and 3 only
D. 1 and 2 only

Correct Answer : B

Answer Justification :

Bharat Stage VI Norms is based on EU Standards. So statement 3 is incorrect.

87 India is planned/planning to sign Military Logistics Support Agreements with which of the following
countries?
1. United States
2. Japan
3. China
4. Australia

Select the correct answer using the code given below


A. 2 and 4 only
B. 1 and 2 only

55
Total Marks : 200
Online Prelims TEST - 19 (SUBJECT WISE)
( InsightsIAS Mock Test Series for UPSC Preliminary Exam 2020 )

C. 1, 2, 3 and 4
D. 1, 2 and 4 only

Correct Answer : D

Answer Justification :

India is looking to conclude three Military Logistics Support Agreements (MLSA). These will further
enhance the operational reach of the military in the region.

India has already signed such agreements with a few countries beginning with the U.S.A.
Agreements with Australia, Japan and Russia are in advanced stages.

88 Afghanistan shares its land border with which of the following countries?
1. India
2. Iran
3. Kazakhstan
4. China

Select the correct answer using the code given below


A. 2 only
B. 1 and 2 only
C. 2, 3 and 4 only
D. 1, 2 and 4 only

Correct Answer : D

Answer Justification :

56
Total Marks : 200
Online Prelims TEST - 19 (SUBJECT WISE)
( InsightsIAS Mock Test Series for UPSC Preliminary Exam 2020 )

89 Consider the following statements regarding Deep Ocean Mission


1. Major thrust of the mission will be looking for deep sea creatures
2. India has been allotted a site of 75,000 sq. km. in the Central Indian Ocean Basin.

Which of the statements given above is/are correct?


A. 1 only
B. 2 only
C. Both 1 and 2
D. Neither 1 nor 2

Correct Answer : B

Answer Justification :

A major thrust of the mission will be looking for metals and minerals. Hence Statement 1 is
incorrect.

India has been allotted a site of 75,000 sq. km. in the Central Indian Ocean Basin (CIOB) by the UN
International Sea Bed Authority for exploitation of polymetallic nodules (PMN). Hence Statement
2 is correct.

https://www.thehindu.com/news/national/centre-to-launch-deep-ocean-mission-in-october/article287
33519.ece

57
Total Marks : 200
Online Prelims TEST - 19 (SUBJECT WISE)
( InsightsIAS Mock Test Series for UPSC Preliminary Exam 2020 )

90 Which of the following is/are functions of universal service obligation fund?


1. Designing the bidding process and carry out the tendering
2. Designing an intelligent subsidy support model underlying the scheme/project.
3. Suggesting such changes in policy as may be deemed necessary for implementation of Universal
Service Support

Select the correct answer using the code given below


A. 2 and 3 only
B. 3 only
C. 1 and 3 only
D. 1, 2 and 3

Correct Answer : D

Answer Justification :

All the statements given above is/are correct.

The New Telecom Policy (NTP) 1999 of Department of Telecom, GoI had Universal Service as one of
its main objectives, as reproduced below -

Strive to provide a balance between the provision of Universal Service to all uncovered areas,
including the rural areas, and the provision of high-level services capable of meeting the needs of
the country's economy and encourage development of telecommunication facilities in remote, hilly
and tribal areas of the country.

The NTP 1999 provided that the resources for meeting the Universal Service Obligation (USO) were
to be generated through a Universal Access Levy (UAL), at a prescribed percentage of the revenue
earned by the telecom licensees to be decided in consultation with the Telecom Regulatory
Authority of India (TRAI).

Functions

Implementation of the guidelines laid down by Government for providing Universal Service
Support;

Suggesting such changes in policy as may be deemed necessary for implementation of


Universal Service Support;

Formulating USOF projects/schemes under the various streams provided in the Indian
Telegraph Rules, 1951, in consultation with telecom service providers and various
stakeholders;

Designing an intelligent subsidy support model underlying the scheme/project, for


reducing/closing the Viability Gap in provisioning of telecommunication service;

58
Total Marks : 200
Online Prelims TEST - 19 (SUBJECT WISE)
( InsightsIAS Mock Test Series for UPSC Preliminary Exam 2020 )

Determining desirable subsidy level, structure & disbursement schedule after undertaking a
suitable costing & modeling/benchmarking exercise and competitive tendering process;

Designing the bidding process and carry out the tendering;

Entering into aptly drafted Agreements with the Universal Service Providers (USPs) with
incentives & disincentives to ensure the achievement of the deliverables;

Monitoring the implementation of USOF projects/schemes and to disburse subsidy in


accordance with the terms & conditions of the respective USOF Agreement;

Accurate & timely settlement of all claims of subsidy/financial support in respect of USOF
schemes

Designing the format of various records and return to be maintained by the USPs;

Carrying out post-implementation review of USOF projects/schemes;

Budgeting and Audit of USOF activities;

Interfacing with international organizations such as ITU, APT and USO funds of other nations;

Forecasting the requirement of Universal Service Funds for each financial year and obtaining
approval of Government through Department of Telecom;

Ensuring that the prescribed Universal Service Levy is credited to the appropriate Universal
Service Fund on a regular basis.

http://www.usof.gov.in/usof-cms/usof-function.jsp

91 Consider the following statements regarding National Company Law Appellate Tribunal (NCLAT)
1. It was constituted under SEBI Act
2. It is an appellate tribunal for hearing appeals against the orders passed by Insolvency and
Bankruptcy Board of India

Which of the statements given above is/are correct?


A. 1 only
B. 2 only
C. Both 1 and 2
D. Neither 1 nor 2

59
Total Marks : 200
Online Prelims TEST - 19 (SUBJECT WISE)
( InsightsIAS Mock Test Series for UPSC Preliminary Exam 2020 )

Correct Answer : B

Answer Justification :

National Company Law Appellate Tribunal (NCLAT) was constituted under Section 410 of the
‘Companies Act, 2013’ for hearing appeals against the orders of National Company Law Tribunal
(NCLT), with effect from 1st June, 2016. Hence Statement 1 is incorrect.

It is also the Appellate Tribunal for hearing appeals against the orders passed by NCLT(s) under
Section 61 of the Insolvency and Bankruptcy Code, 2016 (IBC), with effect from 1st December,
2016.

It is also the Appellate Tribunal for hearing appeals against the orders passed by Insolvency and
Bankruptcy Board of India under Section 202 and Section 211 of IBC. Hence Statement 2 is
correct.

It is also the Appellate Tribunal to hear and dispose of appeals against any direction issued or
decision made or order passed by the Competition Commission of India (CCI), as per the
amendment brought to Section 410 of the Companies Act, 2013 by Section 172 of the Finance Act,
2017, with effect from 26th May, 2017.

92 Consider the following statements


1. Strait of Gibraltar is a narrow strait that connects the Atlantic Ocean to the Mediterranean Sea.
2. The Strait of Hormuz is a strait between the Persian Gulf and the Gulf of Oman.

Which of the statements given above is/are correct?


A. 1 only
B. 2 only
C. Both 1 and 2
D. Neither 1 nor 2

Correct Answer : C

Answer Justification :

Both the statements are correct.

The Strait of Gibraltar is a narrow strait that connects the Atlantic Ocean to the
Mediterranean Sea and separates Gibraltar and Peninsular Spain in Europe from Morocco in
Africa. The two continents are separated by 14.3 kilometres of ocean at the Strait's narrowest point.

The Strait of Hormuz is a strait between the Persian Gulf and the Gulf of Oman. It provides
the only sea passage from the Persian Gulf to the open ocean and is one of the world's most
strategically important choke points.

93 Operation Samadhan, sometime seen in the news, is related to

60
Total Marks : 200
Online Prelims TEST - 19 (SUBJECT WISE)
( InsightsIAS Mock Test Series for UPSC Preliminary Exam 2020 )

1. Strategy to counter Naxalism

A. Strategy to counter Naxalism


B. Prevention strategy to curb the entry of youth into ISIS
C. Government strategy to eliminate terrorism in North Eastern India
D. None of the above

Correct Answer : A

Answer Justification :

Operation ‘SAMADHAN’ is the Ministry of Home Affairs (MHA)’s to counter the Naxal problem

https://pib.gov.in/newsite/PrintRelease.aspx?relid=161625

94 Consider the following statements regarding Dracaena Cambodiana


1. It is an important medicinal plant as well as an ornamental tree.
2. Several antifungal and antibacterial compounds, antioxidants have been extracted from various
parts of the plant.

Which of the statements given above is/are correct?


A. 1 only
B. 2 only
C. Both 1 and 2
D. Neither 1 nor 2

Correct Answer : C

Answer Justification :

Both the statements given above are correct.

Dracaena Cambodiana is an important medicinal plant as well as an ornamental tree.

It is a major source of dragon’s blood, a precious traditional medicine in China. Several antifungal
and antibacterial compounds, antioxidants, flavonoids, etc., have been extracted from various parts
of the plant.

https://www.thehindu.com/sci-tech/science/indias-first-dragon-blood-oozing-tree/article28701517.ec
e

95 Silver Line project, sometime seen in the news, is related to

A. It is a proposal of the Karnataka government that aims to connect major districts and towns
with high speed cyber line.

61
Total Marks : 200
Online Prelims TEST - 19 (SUBJECT WISE)
( InsightsIAS Mock Test Series for UPSC Preliminary Exam 2020 )

B. It is a proposal of the Kerala government that aims to connect major districts and towns with
rapid bus transit system.
C. It is a proposal of the Kerala government that aims to connect major districts and towns with
semi high-speed trains.
D. None of the statements given above (a), (b) and (c) are correct.

Correct Answer : C

Answer Justification :

The Silver Line project is a proposal of the Kerala government that aims to connect major
districts and towns with semi high-speed trains that will run on their own tracks.

Ministry of Railways have recently granted in-principle approval for the project.

It involves laying the railway lines from Kasaragod in the north to Thiruvananthapuram in the south.

96 Consider the following statements regarding Nilgiri Tahr


1. It is critically endangered species
2. It is state animal of Kerala
3. Eravikulam National Park in Kerala is home to the largest population

Which of the statements given above is/are correct?


A. 2 and 3 only
B. 2 only
C. 3 only
D. 1 and 3 only

Correct Answer : C

Answer Justification :

Nilgiri Tahr is an endangered wild goat located in Kerala and parts of Tamil Nadu. It is state animal
of Tamil Nadu. Hence Statement 1 and 2 are incorrect.

It is listed in Schedule I of Wildlife (Protection) Act, 1972 and as Endangered on IUCN Red List.

Eravikulam National Park in Kerala is home to the largest population. Hence Statement 3 is


correct.

97 Global Gender Gap Index, measure progress towards parity between men and women in which of the
following parameters
1. Economy
2. Education
3. Health

62
Total Marks : 200
Online Prelims TEST - 19 (SUBJECT WISE)
( InsightsIAS Mock Test Series for UPSC Preliminary Exam 2020 )

4. Political representation.

Select the correct answer using the code given below


A. 2 and 3 only
B. 1, 2 and 3 only
C. 2, 3 and 4 only
D. 1, 2, 3 and 4

Correct Answer : D

Answer Justification :

The report examines four overall areas of inequality between men and women in 130 economies
around the globe, over 93% of the world's population:

Economic participation and opportunity – outcomes on salaries, participation levels and


access to high-skilled employment

Educational attainment – outcomes on access to basic and higher level education

Political empowerment – outcomes on representation in decision-making structures

Health and survival – outcomes on life expectancy and sex ratio. In this case parity is not
assumed, there are assumed to be fewer female births than male (944 female for every 1,000
males), and men are assumed to die younger. Provided that women live at least six percent
longer than men, parity is assumed. But if it is less than six percent it counts as a gender gap.

98 Consider the following statements regarding Protection of Children from Sexual Offences Act, 2012
(POSCO)
1. Police should contact Child Welfare Committee (CWC) within 24 hours of report of abuse
2. The case must be disposed of within one year
3. POSCO has made gender neutral

Which of the statements given above is/are correct?


A. 2 only
B. 1 and 3 only
C. 1, 2 and 3
D. 3 only

Correct Answer : C

Answer Justification :

63
Total Marks : 200
Online Prelims TEST - 19 (SUBJECT WISE)
( InsightsIAS Mock Test Series for UPSC Preliminary Exam 2020 )

In order to effectively address the heinous crimes of sexual abuse and sexual exploitation of
children through less ambiguous and more stringent legal provisions, the Ministry of Women and
Child Development championed the introduction of the Protection of Children from Sexual Offences
(POCSO) Act, 2012.

Salient features

The Act defines a child as any person below eighteen years of age, and regards the best
interests and well-being of the child as being of paramount importance at every stage, to
ensure the healthy physical, emotional, intellectual and social development of the child.

It defines different forms of sexual abuse, including penetrative and non-penetrative assault,
as well as sexual harassment and pornography, and deems a sexual assault to be
“aggravated” under certain circumstances, such as when the abused child is mentally ill or
when the abuse is committed by a person in a position of trust or authority vis-à-vis the child,
like a family member, police officer, teacher, or doctor.

People who traffic children for sexual purposes are also punishable under the provisions
relating to abetment in the Act. The Act prescribes stringent punishment graded as per the
gravity of the offence, with a maximum term of rigorous imprisonment for life, and fine.

POSCO has made gender neutral.

Police should contact Child Welfare Committee (CWC) within 24 hours of report of abuse. The
case must be disposed of within one year.

http://vikaspedia.in/education/policies-and-schemes/protection-of-children-from-sexual-offences-act

99 Consider the following statements regarding National Digital Communications Policy 2018
1. It aims to provide broadband access to every citizen at 1000 Mbps speed by the year 2022.
2. It aims at creating at least 40 lakh new jobs in the sector by 2022
3. It aims to propel India to the Top 50 nations in the ICT Development Index.

Which of the statements given above is/are correct?


A. 1 and 2 only
B. 2 and 3 only
C. 1 and 3 only
D. 1, 2 and 3

Correct Answer : B

Answer Justification :

64
Total Marks : 200
Online Prelims TEST - 19 (SUBJECT WISE)
( InsightsIAS Mock Test Series for UPSC Preliminary Exam 2020 )

It aims to provide broadband access to every citizen at 50 Mbps speed by the year 2022. Hence
Statement 1 is incorrect.

National Digital Communications Policy - 2018 has been formulated, in place of the existing
National Telecom Policy-2012, to cater to the modern needs of the digital communications sector of
India.

Objectives

The key objectives of the policy are:

Broadband for all;

Creating four million additional jobs in the Digital Communications sector;

Enhancing the contribution of the Digital Communications sector to 8% of India's GDP from ~
6% in 2017;

It aims at creating at least 40 lakh new jobs in the sector by 2022. Hence Statement 2 is
correct.

Propelling India to the Top 50 Nations in the ICT Development Index of ITU from 134
in 2017; Hence Statement 3 is correct.

Enhancing India's contribution to Global Value Chains; and

Ensuring Digital Sovereignty.

http://vikaspedia.in/e-governance/digital-india/national-digital-communications-policy-2018

100 Gender Inequality Index, sometime seen in the news, is released by

A. World Bank
B. United Nations Development Programme
C. World Economic Forum
D. UN Women

Correct Answer : B

Answer Justification :

65
Total Marks : 200
Online Prelims TEST - 19 (SUBJECT WISE)
( InsightsIAS Mock Test Series for UPSC Preliminary Exam 2020 )

The Gender Inequality Index (GII) is an index for measurement of gender disparity that was
introduced in the 2010 Human Development Report 20th anniversary edition by the United Nations
Development Programme (UNDP).

A initiative to provide watermarks and bookmarks free pdfs to you.


Share and Subscribe our telegram channel
@visionpt3652019

https://t.me/visionpt3652019

66

You might also like